Gastro 18 Banco Cont

puep enarmDescripción completa

Views 167 Downloads 4 File size 1MB

Report DMCA / Copyright

DOWNLOAD FILE

Recommend stories

Citation preview

DIAGNÓSTICO CLÍNICO CASO 1 Mujer de 32 años de edad, acude a consulta por pirosis diaria. Está en tratamiento con pantoprazol 40 mg 30 min antes del desayuno desde hace 4 semanas y no ha notado mejoría clínica. Niega náuseas, vómito, disfagia y pérdida de peso. La conducta a seguir es: Solicitar panendoscopia Incrementar dosis de Inhibidores de la bomba de protones Solicitar pHmetría Agregar ranitidina La pirosis es uno de los síntomas cardinales de Enfermedad por reflujo gastroesofágico. Está indicado realizar estudios de extensión cuando no hay una respuesta adecuada a IBP con dosis doble. En este caso la paciente tiene monodosis por lo que no es posible catalogarlo como ERGE refractario. Tampoco tiene datos de alarma.

Después de haber tomado pantoprazol dos veces al día, continúa con misma sintomatología. Usted decide: Solicitar pHmetría Solicitar panendoscopia Realizar esofagograma Agregar antiácidos En ERGE refractario está indicado realizar panendoscopia para descartar otras causas.

El resultado del estudio fue normal. Con lo anterior usted decide: Solicitar pHmetría Solicitar manometría esofágica Solicitar esofagograma Enviar a cirugía No tiene evidencia de reflujo erosivo, por lo que es necesario hacer pHmetría para ver si existe reflujo patológico y descartar otras patologías como pirosis funcional.

El estudio fue reportado como normal. El diagnóstico más probable en esta paciente es: Enfermedad por reflujo gastroesofágico refractaria a tratamiento Pirosis funcional Trastorno psiquiátrico Ninguna de las anteriores ¿Cuál de los siguientes tratamientos es apropiado para esta paciente? Omeprazol a altas dosis Antidepresivo tricíclico Ranitidina nocturna y omeprazol Antiácidos

Los antidepresivos tricíclicos se utilizan en el tratamiento de la pirosis funcional por desensibilizar el esófago.

CLÍNICO CASO 2 Hombre de 60 años de edad con antecedente de tabaquismo con IT de 25 e ingesta abundante de alcohol (40 gramos al día). Inició hace 8 semanas con evacuaciones diarreicas en cantidad aproximada de 5 a 6 al día, con esteatorrea y lientería. Las evacuaciones disminuyen con el ayuno y casi no se levanta durante la noche al baño a menos que su último alimento haya sido copioso. Durante este período ha perdido aproximadamente 3 kg de peso. De acuerdo a sus características clínicas ¿Cómo clasificaría la diarrea? Acuosa Malabsortiva Inflamatoria Funcional Se trata de una diarrea malabsortiva (tiene esteatorrea y lientería), mejora con el ayuno y empeora con la alimentación.

¿Cuál de los siguientes no sería un diagnóstico diferencial en este caso? Atrofia de vellosidades intestinales Deficiencia de lactasa Sobrepoblación bacteriana Insuficiencia pancreática exocrina Todas las opciones comentadas ocasionan diarrea malabsortiva ya sea originada en la pared intestinal o en el lumen intestinal. La deficiencia de lactasa no tiene por qué cursar con malabsorción de grasas y ocasionar esteatorrea.

Dentro del abordaje se realizó una tomografía de abdomen que mostró calcificaciones difusas en el parénquima pancreático y dilatación del conducto pancreático hasta 5 mm. ¿Cuál es la causa más frecuente de este padecimiento? Consumo de alcohol Aumento en la cantidad de bacterias no patógenas en el intestino delgado Cambios en el microbioma intestinal Reacción alérgica al gluten La causa más frecuente de pancreatitis crónica calcificante es el consumo de alcohol.

¿Cómo esperaría encontrar el resultado de la prueba de D-xilosa y por qué motivo? Normal – se requiere pérdida del 90% de la función exocrina del páncreas para alterar este resultado Anormal – Con alteraciones sutiles de la función pancreática este resultado se altera

Normal – La D-xilosa requiere únicamente una pared intestinal íntegra para su absorción Normal – La D-xilosa se absorbe en el colon En este caso la diarrea parece por pancreatitis crónica (insuficiencia pancreática exocrina). La D-xilosa es un monosacárido que únicamente requiere integridad de la pared intestinal para su absorción. En pacientes con insuficiencia pancreática y una pared intestinal íntegra, este resultado no tiene por qué alterarse.

¿Cuál es el tratamiento de elección en este caso? Tetraciclinas por 6 meses Enzimas pancreáticas Colestiramina Descompresión endoscópica del conducto pancreático El tratamiento de elección para la insuficiencia pancreática exocrina son las enzimas pancreáticas. Los tratamientos derivativos tanto quirúrgico como endoscópico en pancreatitis crónica, se utilizan para el tratamiento del dolor.

CLÍNICO CASO 3 Mujer de 24 años de edad sin antecedentes de relevancia. Inició hace 4 años con episodios de constipación caracterizada por heces duras y en escíbalos acompañadas de dificultad para evacuar, en ocasiones acompañadas de tenesmo. Niega otros síntomas. ¿Cuál diagnóstico diferencial es el menos probable? Constipación de tránsito colónico lento Trastorno defecatorio con disinergia Constipación funcional Síndrome de intestino irritable variedad constipación Para integrar el diagnóstico de síndrome de intestino irritable es necesario que el paciente tenga dolor abdominal y en este caso no hay dolor abdominal. El resto de las opciones son formas de constipación primaria o idiopática y son diagnósticos diferenciales en este caso. Se requeriría mayor información para inclinarse por algún diagnóstico específico.

¿Cuál es el tratamiento de elección inicial en este caso? Polietilenglicol Fibra Lactulosa Antiespasmódicos El tratamiento inicial de la constipación es fibra seguida por laxantes de preferencia osmóticos.

En caso de que no mejorara con el tratamiento indicado y de acuerdo a las características clínicas. ¿Cuál es el abordaje diagnóstico a seguir en este caso? Colonoscopia Manometría anorrectal Rectosigmoidoscopia Enema baritado El primer paso en el abordaje de constipación es un buen interrogatorio y exploración para descartar causas secundarias. El abordaje debe ser

individualizado, debe evitarse un abordaje extenso con múltiples estudios innecesarios. Esta paciente no tiene datos de alarma y no hay algo que oriente que requiera estudios invasivos para su abordaje. El siguiente paso para el abordaje diagnóstico en una constipación primaria es la manometría anorrectal.

El estudio previamente indicado por usted se reportó normal. ¿Cuál es el siguiente paso en el abordaje diagnóstico? Rectosigmoidoscopia Enema baritado Tránsito colónico Tránsito intestinal Una vez que se descartaron trastornos defecatorios mediante manometría ano-rectal. El siguiente paso en el abordaje de la constipación primaria o idiopática, es medir el tránsito colónico. Esto se logra mediante marcadores radio-opacos que se ingieren en cápsulas. Existen múltiples protocolos validados para clasificar si el tránsito colónico normal o es lento.

¿Cuál de los siguientes tratamientos de constipación ocasiona melanosis coli? Senósidos Polietilenglicol Lactulosa Psyllium Plantago Los laxantes estimulantes se asocian a melanosis coli y colon catártico. En algunos casos, el hallazgo colonoscópico de melanosis coli orienta al abuso de laxantes estimulantes.

CLÍNICO CASO 4 Mujer de 75 años de edad diabética de larga evolución sin complicaciones crónicas, con arritmia cardíaca no especificada que nunca le ha condicionado síntomas. Inició hace 2 horas con dolor abdominal mesogástrico intenso, acompañado de náusea y vómito. A la exploración física se encuentra intranquila, diaforética, con facies álgica. Taquicárdica con FC de 100, se escucha ritmo cardíaco irregular, FR 20, TA 140/80mmHg, temperatura de 36 °C. La exploración abdominal destaca solo por disminución discreta de los ruidos peristálticos en intensidad y frecuencia. No hay datos de irritación peritoneal. Se solicitó electrocardiograma que mostró ritmo “irregularmente irregular” con ausencia de ondas P. La radiografía de abdomen no mostró alteraciones. ¿Cuál de los siguientes sería su primer estudio diagnóstico? Ultrasonido abdominal Resonancia magnética de abdomen Tomografía de abdomen contrastada Tomografía de abdomen no contrastada

Es un caso probable de isquemia mesentérica donde los hallazgos a la EF son desproporcionales a la intensidad del dolor del paciente. El primer paso en el abordaje diagnóstico es la TAC contrastada de abdomen o si es posible y está al alcance una angiotomografía de abdomen.

El estudio solicitado mostró un defecto de llenado en la arteria mesentérica superior. No se observa gas venoso portal ni neumatosis intestinal. ¿Cuál de las siguientes es la etiología más probable en esta paciente? Trombosis arterial mesentérica in situ Embolismo a la arteria mesentérica superior Trombosis venosa mesentérica Variante anatómica congénita De acuerdo al caso presentado la paciente parece tener fibrilación auricular. Esta entidad particularmente en pacientes de edad avanzada, con diabetes y con otros factores de riesgo que se encuentran en el acrónimo CHADS-VASC, tienen riesgo de trombosis en la orejuela del atrio izquierdo con ulterior embolismo. El riesgo trombótico incrementa a mayor número de factores de riesgo.

¿Cuál es el sitio anatómico más frecuentemente afectado en estos casos? Ostium de la arteria mesentérica superior Ostium del tronco celíaco Arteria mesentérica superior distal a arteria cólica media Ostium de la arteria mesentérica Un 85% de los émbolos se aloja distal a la arteria cólica media.

¿Cuál de los siguientes es un sitio intestinal de susceptibilidad a la isquemia? Ángulo hepático Ángulo esplénico Ciego Íleon terminal El ángulo esplénico (Griffith) y la unión recto-sigmoidea (Sudeck), son sitios de susceptibilidad a la isquemia debido a que son áreas que comparten circulación de arteria mesentérica superior con inferior.

¿Cuál es el paso siguiente en el tratamiento de la paciente? Anticoagulación y LAPE Anticoagulación y angiografía Esplenoportografía Laparotomía exploratoria La paciente no tiene datos de indicación quirúrgica urgente como son datos de irritación peritoneal y los estudios de imagen no muestran gas venoso portal ni neumatosis intestinal. Además se encuentra descrito que el tiempo de viabilidad y de ventana terapéutica en las isquemias mesentéricas arteriales agudas es de aproximadamente 12 horas.

CLINICAL CASE 5 A 50-year-old man with diabetes mellitus presents with melena and microcytic anemia. He reports having epigastric pain in the last week. His only medication at this moment is metformin and insulin. Upon admission to the hospital his BP is 90/50 mmHg and HR 110 beats per minute. His hemoglobin is 8.5 mg/dL and INR 1.3. What is the most appropriate step in the management of this patient? Nasogastric lavage to evaluate for upper GI blood loss Administration of a proton pump inhibitor Administration of intravenous fluids Urgent endoscopy La administración de IBP forma parte del manejo en sangrado de tubo digestivo alto, sin embargo el paciente tiene datos que sugieren hipovolemia (hipotensión y taquicardia), por lo que el manejo inicial en este caso es reanimación con líquidos intravenosos.

After 2 hours, his vital signs are stable. What is the next step? Transfusion of fresh frozen plasma Prophylactic administration of antibiotics Hospital discharge with PPI therapy Upper gastrointestinal endoscopy De acuerdo con sus estudios de laboratorios el paciente no tiene requerimientos transfusionales. El paciente amerita estudio de endoscopia para localizar el sitio de sangrado.

An upper endoscopy reveals a clean based ulcer in the duodenum. According to the result, which of the following is the most likely etiology? Surreptitious use of non-steroidal anti-inflammatory drugs Helicobacter pylori infection Malignant peptic ulcer Zollinger-Ellison syndrome La causa más frecuente de úlcera duodenal es infección por H pylori (hasta 90% de los casos)

Gastric biopsies were taken and the histopathology demonstrated the presence of Helicobacter pylori. Then you decide: Dual therapy: PPI plus sucralfate Continue PPI therapy for four weeks Empiric therapy for Helicobacter pylori Culture-guided therapy for H. pylori Amerita tratamiento con IBP durante 8 semanas para lograr la curación completa de la úlcera. Toda úlcera asociada a H. pylori amerita tratamiento de erradicación. Si es el primer esquema se opta por tratamiento empírico. El tratamiento guiado con cultivo está indicado en falla al tratamiento.

The pathogenesis of peptic ulcer involves an imbalance between defense and aggravating factors. Which of the following can both damage the gastric mucosal barrier and stimulate gastric acid secretion? Gastric Mucous Helicobacter pylori Non-steroidal anti-inflammatory drug H. pylori produce amonio, el cual produce daño directo a la mucosa gástrica. Por otro lado puede incrementar de forma transitoria la secreción de HCl por hiperplasia de las células G.

CLINICAL CASE 6 A 60 year old woman with a history of hysterectomy because of uterine leiomyoma’s, presents to the emergency department. She reports severe abdominal pain with biliary vomiting for the last 2 hours. Physical examination reveals a woman in acute distress, her vital signs are normal, except for mild tachycardia (90 bpm). Abdomen examination reveals increased bowel sounds with a metallic pitch. There are no peritoneal signs. What’s the best next step in the diagnostic approach? Abdominal USG Abdominal RX Abdominal CT Laparotomy The first step in the diagnostic approach of a bowel obstruction is abdominal RX.

Which of the options would be discordant with the clinical picture? Presence of air in the transverse colon Absence of air in the colon Presence of air fluid levels Presence of a transition zone This clinical picture is typical of mechanical acute bowel obstruction (history of abdominal surgery, increased bowel sounds with metallic pitch). It’s not usual to find colonic gas in mechanical obstructions in contrast to paralytic ileus.

Which of the next options is not appropriate in the management of this patient? Nasogastric tube High dose opiates Hydration Abdominal CT Opiates may contribute to further obstruction because their effects in GI motility.

Which is the most common cause of the patient clinical picture? Postoperative ileus Postoperative intestinal adhesions Hypokalemia Medications The most common cause of mechanical obstruction is postoperative adhesions.

The patient had initial improvement with your management with further deterioration after 48 hours. What’s the best next step in the management? Laparotomy Colonoscopy decompression Upper GI endoscopy Continue with established management until 5 days have elapsed After 48 to 72 hours of no improvement with conservative management, the next step in the management of a mechanical bowel obstruction is surgery.

CLINICAL CASE 7 A 25 year old woman presents to your office with a 3 month history of 2-3 bloody bowel movements per day associated with tenesmus. Physical examination is unremarkable. You decide to order stool cultures and ova / parasites, getting with negative results. Which of the following is the best next step in the diagnostic approach? Watch and wait Colonoscopy Abdominal CT Abdominal MRI The initial step in the approach of a chronic inflammatory diarrhea as in this patient is colonoscopy or at least rectosigmodoscopy.

The diagnostic test that you ordered ruled out tumors and vascular causes in the patient’s clinical picture. According to this, what’s your first differential diagnosis at this moment? Anal fissure Internal hemorrhoids Ulcerative colitis Intestinal vasculitis The clinical picture is typical of ulcerative colitis. After ruling out important causes of lower GI bleeding in a patient with this features, UC is one of our first differential diagnosis.

Which of the following describes the most probable findings in the study ordered? Flask shaped ulcers scattered throughout the colon Erythema, friability and ulcers predominantly at the transverse colon Erythema friability and ulcers starting in the rectum and expanding towards proximal colon Complex perianal fistulae

Abdominal USG revealed a nodular liver. You decide to order a Fibro scan with reveals fibrosis F4. Which of the following is not appropriate in the management of this patient? Lower GI endoscopy Upper GI endoscopy Periodic USG (every 6 months) Immunization for HAV, HBV

The endoscopic features of UC are those listed and highlighted. UC affects exclusively the colonic epithelium. Crohn’s disease affects the whole intestinal wall.

Lower GI endoscopy has no role in this patient. He has no history of lower GI bleeding and there are no other indications for this test at this moment.

In mild forms of this disease, what’s the first choice treatment? Infliximab Mesalamine Steroids Methotrexate Amino salicylates in their oral or topical forms are useful in all forms of UC. In mild forms of the disease, sometimes is enough treatment with this drug class to control disease activity.

Which of the following conditions is not associated with this disease? Pyoderma gangrenous Uveitis Primary sclerosing cholangitis Duodenal cancer Duodenal cancer is not associated with ulcerative colitis. A colonic condition associated with duodenal cancer is familiar adenomatous polyposis.

CLINICAL CASE 8 A 31 year old man with long-standing HCV infection presents to your office with progressive fatigue and lower extremity edema. Physical examination reveals mild edema around the heels. His fibrosis status is unknown. Apart from general laboratory tests, which of the following is the next first step in the diagnostic approach? Liver biopsy Abdominal USG Abdominal CT Upper GI endoscopy The first step in the approach to this patient is USG. USG is a non-invasive and inexpensive way to know the main features of liver anatomy, liver borders, its blood vessels and the presence or absence of splenomegaly. Right now, fibro scan and other non-invasive methods for measuring liver fibrosis are becoming more frequently used for this purpose. Even though liver biopsy remains as the gold standard for diagnosis of liver fibrosis it’s invasive, has significant costs and complications, so is not commonly used as a first test.

One of the studies revealed varices “at some place of his gut”. He doesn’t remember where. He remember that the report revealed presence of red-spots among big varices. The patient does not have a history of GI bleeding. Which of the following options is appropriate at this point for the management? Variceal band ligation + propranolol Propranolol only Wait until further decompensation occurs to intervene Metoprolol only There are two choices for primary prophylaxis of variceal bleeding, both with similar efficacy: non-selective beta blockers or variceal band ligation. But at this point, there’s no role for combination therapy in primary prophylaxis (Variceal band ligation + beta blocker). Although sclerotherapy was used for this purpose, we have more effective and safer options.

What’s the main imaging feature of primary hepatic malignancies in patient with cirrhosis? Hypo dense lesion without contrast enhancement Enhancement in arterial phase with washout in latter phases Enhancement in portal venous phase Ring enhancement Hepatocellular carcinoma in cirrhotic patients, can be diagnosed without biopsy, as long as the imaging features are typical as highlighted in the above option.

In patients with cirrhotic ascites, which of the following options is not appropriate? Paracentesis at first presentation Paracentesis in hospitalized patients with ascites Paracentesis only if there are clinical clues of infection Large volume paracentesis with IV albumin form patients with tense ascites In general the threshold to obtain a sample from ascites in patient with cirrhosis is low. There are many cases of spontaneous bacterial peritonitis without clinical features of infection.

CLINICAL CASE 9 40 year old man with intermittent dysphagia to both solids and liquids, and retrosternal pain, with no

regurgitation. A cardiologist discarded ischemic cardiomyopathy. In this case you should suspect: Mechanic disorder Motor disorder Stenosis Psychiatric disease La disfagia intermitente y a líquidos orienta hacia causas motoras.

What kind of dysphagia does this patient have? Esophageal Oropharyngeal Mechanic Neurologic La disfagia orofaríngea usualmente se acompaña de tos, aspiración o dificultad para la propulsión del bolo alimenticio, por lo tanto este paciente tiene disfagia esofágica.

Which of these options is not routinely employed during dysphagia approach? Manometry Upper endoscopy Abdominal tomography Upper GI series La TAC no tiene papel en el abordaje de la disfagia. Sólo en pacientes con neoplasias diagnosticadas tiene utilidad en el estadiaje.

A manometry showed contractions of 180mmHg, with simultaneous contractions in 10% of wet swallows and lower esophageal sphincter pressure of 20mmHg with residual pressure of 5mmHg. The probable diagnosis is: Hypertensive lower esophageal sphincter pressure Nutcracker esophagus Achalasia Cricopharyngeal achalasia Esófago en cascanueces tiene contracciones enérgicas con >180 mmHg, dolor y disfagia.

First line treatment for the upper condition includes the following: PPI therapy and calcium channel blockers Butylhyoscine Myotomy Cricopharyngeal myotomy Tratamiento con IBP y relajantes de musculo liso. Miotomía en casos extremos.

OTHER QUESTIONS The proenzyme pepsinogen is secreted mainly from which of the following structures: I cells of the duodenum G cells of the stomach

Chief cells of the stomach Principal cells of the stomach El pepsinógeno es producido en el estómago por las células principales de las glándulas oxínticas localizadas en el fondo gástrico y cuerpo gástrico.

Which of the following is likely consequence of ileal resection? Achalasia Vitamin B12 deficiency Atrophic gastritis Peptic ulcer La vitamina B12 es absorbida a nivel del íleon distal. Por lo tanto, cualquier patología que afecte el íleon distal (resección quirúrgica, enfermedad de Crohn) puede cursar deficiencia de esta vitamina.

Which of the following structures undergoes receptive relaxation when a bolus of food is swallowed? Pharynx Upper esophageal sphincter Esophagus Stomach La relajación receptiva permite que el estómago acomode un mayor contenido alimenticio dentro.

Which of the following hormones is released by the presence of fat and proteins in the duodenum and has a major effect to decrease gastric emptying? Cholecystokinin (CCK) Gastrin Secretin Motilin CCK y secretina son hormonas secretadas en el duodeno por las células I y S en respectivamente. Las proteínas y grasas estimulan su liberación, pero la CCK es la única que puede retrasar el vaciamiento gástrico.

Various proteolytic enzymes are secreted in an inactive form into the small intestine. Which of these are important for activating one or more proteolytic enzymes, converting them to an active form? Enterocinase Trypsinogen and trypsin Trypsin and enterocinase Trypsin, enterocinase and pepsin La enterocinasa activa de forma inicial al tripsinógeno (proenzima) para producir tripsina la cual puede activar otras moléculas de tripsinógeno. (Fenómeno de cascada)

Fisiología gastrointestinal / Disfagia y trastornos de motilidad esofágica / Enfermedad por Reflujo Gastroesofágico CASO CLÍNICO 1 Mujer de 75 años de edad con diagnóstico de artritis reumatoide, consume naproxeno de forma regular

para control del dolor. Hace una semana presentó melena. Usted decide realizar una panendoscopia en la que se observó una úlcera gástrica recubierta de fibrina. ¿Cuál es la localización más frecuente? Curvatura mayor Fondo Antro Incisura Sitio más frecuentes son antro y curvatura menor

De acuerdo con la clasificación de Forrest es: Forrest Ib Forrest IIa Forrest IIc Forrest III Ia y Ib son con sangrado activo. IIa y IIb son con vaso visible y coágulo adherido. Ulcera recubierta de fibrina o base limpia es Forrest III.

El tratamiento de esta úlcera es el siguiente: Vigilancia Inhibidor de la bomba de protones (IBP) Inyección con adrenalina Sonda bipolar Solo IBP, por ser Forrest III no amerita tratamiento endoscópico.

¿Cuál sería el manejo a largo plazo en esta paciente? Vigilancia Suspender naproxeno Naproxeno + IBP Cambiar a otro antiinflamatorio no esteroideo Es necesario suspender el agente causal que en este caso son los AINES.

¿Qué porcentaje de las úlceras gástricas son asociadas a H. pylori? 90% 80% 60% 30% El 60% de las ulceras gástricas se asocian a H pylori y el 90% de las duodenales

CASO CLÍNICO 2 Mujer de 35 años de edad con anemia por deficiencia de vitamina B12, se le solicitaron anticuerpos anticélulas parietales que resultaron positivos. Se le realizó panendoscopia en la que se observó atrofia de la mucosa gástrica. Las biopsias mostraron gastritis crónica atrófica sin displasia.

Con estos hallazgos el diagnóstico es: Anemia perniciosa Gastritis atrófica ambiental Gastritis crónica tipo B Gastritis crónica tipo AB La deficiencia de Vitamina B12, gastritis atrófica con anticuerpos positivos hace el diagnóstico de anemia perniciosa.

¿Cuál es la distribución típica en este tipo de gastritis atrófica? Fondo y cuerpo Antro Curvatura menor Cardias Gastritis atrófica tipo A es la que se asocia con anemia perniciosa, y se localiza en cuerpo y fondo.

¿Qué alteración espera encontrar en los estudios de laboratorio? Anemia microcítica Anemia macrocítica Deficiencia de ácido fólico Anemia por deficiencia de hierro Anemia macrocítica (VCM elevado) por la deficiencia de vitamina B12 es lo más característico.

¿Cuál es el tratamiento de esta patología? Hierro Cianocobalamina Piridoxina Tiamina Tratamiento es la reposición de vitamina B12 o cianocobalamina.

Esta patología se ha asociado con: Metaplasia intestinal Tumor epidermoide Úlcera péptica Ulcera intestinal Gastritis crónica predispone a metaplasia intestinal y displasia.

CASO CLÍNICO 3 Mujer de 20 años de edad quien desde hace 3 meses presenta dolor abdominal tipo ardoroso en epigastrio. Ha tomado un inhibidor de la bomba de protones durante 8 semanas sin respuesta. Niega datos de alarma.

El cuadro clínico es compatible con: Dispepsia Gastritis Neoplasia gástrica Síndrome de intestino irritable Usted decide: Realizar tomografía de abdomen Solicitar prueba de aliento Realizar panendoscopia Iniciar antidepresivo Toda dispepsia no investigada amerita prueba de aliento para detección de H. pylori.

Los siguientes, uno no es un dato de alarma en esta patología: Historia familiar de cáncer gástrico Anemia Edad menor a 45 años Vómito Datos de alarma asociados a dispepsia son indicación de panendoscopia como estudio inicial. Estos son historia familiar de cáncer gástrico, anemia, vómito, edad mayor a 55 años, pérdida de peso.

¿Cuál es la etiología más frecuente de esta patología? Fármacos Úlcera péptica Cáncer gástrico Funcional El 70% son funcionales. El 30% tienen etiología orgánica.

Tiene prueba de aliento negativa y panendoscopia normal. De acuerdo con estos resultados, ¿cuál sería el tratamiento de elección? Continuar IBP Tratamiento empírico para H. pylori Sucralfato Antidepresivo El diagnóstico de la paciente es de dispepsia funcional, el tratamiento de elección en dispepsia funcional es antidepresivo.

CASO CLÍNICO 4 Mujer de 35 años de edad con disfagia a sólidos y líquidos de 6 meses de evolución, regurgitación, dolor retroesternal y pérdida de peso. Por las características de los síntomas, la disfagia es: Orofaríngea Faríngea

Esofágica Mecánica El cuadro clínico es compatible con acalasia que es un trastorno motor esofágico.

De los siguientes estudios, ¿cuál no forma parte del abordaje inicial en un paciente con disfagia? Tomografía abdominal Panendoscopia Serie esofagogastroduodenal Manometría Se le realizó una serie esofagogastroduodenal observando dilatación importante del esófago y signo del pico de pájaro. La manometría esofágica mostró falta de relajación del esfínter esofágico inferior y aperistalsis del cuerpo esofágico. ¿Cuál es el diagnóstico de este paciente? Esófago hipercontráctil Obstrucción al flujo de salida Acalasia Espasmo esofágico difuso El patrón manométrico característico de la acalasia es ausencia de relajación del EEI y aperistalsis.

De acuerdo con lo anterior, usted decide: Enviar a cirugía Indicar bloqueadores de canales de calcio Enviar para dilatación endoscópica Realizar una panendoscopia Aunque el esofagograma y la manometría son diagnósticos, es necesario realizar panendoscopia para descartar pseudoacalasia.

La panendoscopia se reportó con esófago dilatado y restos alimenticios. El tratamiento de elección en esta patología es: Miotomía de Heller Toxina botulínica Calcio antagonista Inhibidores de fosfodiesterasa 5 Tratamiento de elección es con miotomía de Heller o dilatación endoscópica.

CASO CLÍNICO 5 Hombre de 75 años de edad con disfagia de 5 meses de evolución al iniciar la deglución. Refiere sensación de atoramiento a nivel del cuello, por lo que toma abundantes líquidos durante las comidas presentando regurgitación intermitente y halitosis.

¿Cuál es el diagnóstico más probable? Disfagia orofaríngea Disfagia esofágica Enfermedad por reflujo gastroesofágico Esófago hipercontráctil La disfagia orofaríngea implica dificultad para iniciar la deglución, la refieren como sensación de atoramiento en el cuello. Suele acompañarse de tos, regurgitación y halitosis.

De acuerdo con su sospecha clínica, ¿qué estudio solicitaría de forma inicial? Panendoscopia pHmetría de 24 horas Manometría esofágica Videofluoroscopia En disfagia orofaríngea el estudio inicial es videofluoroscopia. El resto es para disfagia esofágica.

Un trago de bario demostró la presencia de un divertículo en el tercio superior del esófago. Éste corresponde a: Divertículo epifrénico Divertículo de Zenker Divertículo por pulsión Diverticulosis El divertículo en tercio superior de esófago es el de Zenker y se forma por un mecanismo de pulsión. El epifrénico se localiza en tercio distal del esófago.

¿Cuál es el sitio típico de aparición de este divertículo? Cinco centímetros por debajo del esfínter esofágico superior Triángulo de Killian Cinco centímetros por arriba del esfínter esofágico inferior Puede aparecer en cualquier sitio del esófago ¿Cuál es el tratamiento en este caso? Tratamiento farmacológico Diverticulectomía Dilatación endoscópica Pantoprazol El tratamiento de elección en divertículo de Zenker es resección del divertículo.

CASO CLÍNICO 6 Mujer de 36 años de edad con enfermedad mixta del tejido conectivo. Toma omeprazol de forma regular por pirosis y regurgitación. Acude por disfagia de 4

meses de evolución de predominio a sólidos la cual ha sido progresiva. ¿Qué tipo de disfagia presenta la paciente? Disfagia motora Disfagia orofaríngea Disfagia mecánica Disfasia fisiológica La disfagia que inicia con sólidos es por un problema mecánico principalmente estenosis péptica. La disfagia que inicia con sólidos y líquidos indica problema motor.

¿Qué estudio sería de utilidad en el abordaje de esta paciente? Endoscopia Panendoscopia Manometría esofágica Videofluoroscopia Trago con bario y panendoscopia son de utilidad en disfagia mecánica. La manometría es de utilidad en disfagia de tipo motor.

En la panendoscopia se observó una estenosis esofágica en tercio distal no franqueable con el endoscopio. ¿La etiología más probable en esta paciente es? Estenosis maligna Estenosis péptica Esófago de Barrett Membrana esofágica Pacientes con enfermedad mixta del tejido conectivo cursan con trastornos de la motilidad esofágica y síntomas típicos de reflujo. Debe de tener una estenosis péptica por reflujo.

El tratamiento de elección en esta paciente es: En este momento no amerita tratamiento hasta que los síntomas empeoren Valoración por cirugía Dilatación endoscópica Miotomía Estenosis no franqueable con síntomas amerita tratamiento con dilatación para mejorar la disfagia

Se realizó dilatación por endoscopía y se documentó hernia hiatal de 4 cm. El tratamiento a largo plazo de esta paciente será: Optimizar tratamiento con IBP Enviar a cirugía para funduplicatura Dieta en tercios Bajar cabecera

Por la enfermedad del tejido conectivo es probable que tenga problemas esofágicos motores por lo que no sería una buena candidata para funduplicatura. Su tratamiento es con IBP, dieta en quintos, elevar cabecera, etc.

Dispepsia/ Úlcera péptica/ Gastritis/ Diarrea crónica CASO CLÍNICO 1 Hombre de 28 años de edad tiene cuadros de diarrea con la ingesta de lácteos. ¿A qué tipo de diarrea corresponde? Inflamatoria Secretora Osmótica Motora De acuerdo con lo anterior, ¿cuál es el mecanismo fisiopatológico? Daño e inflamación de la mucosa de colon Secreción de agua y electrolitos Sobrecrecimiento bacteriano Presencia de solutos no absorbibles Principalmente es secundaria a presencia a solutos no absorbibles en el tracto gastrointestinal: la presencia de sustancias altamente osmóticas permiten la salida de agua hacia el lumen disminuyendo la osmolaridad intraluminal y esto disminuye los iones inorgánicos.

La etiología es: Tránsito intestinal acelerado Deficiencia enzimática Menor superficie de absorción Daño intestinal La intolerancia a los lácteos produce diarrea osmótica por solutos no absorbibles por deficiencia de la enzima lactasa.

Es una característica de este tipo de diarrea: Cede con el ayuno No cede con el ayuno Desequilibrio hidroelectrolítico Evacuaciones con moco y sangre Típicamente cede con ayuno lo que la distingue de la secretora que no cede.

¿Cuál de los siguientes estudios puede ayudar a distinguir una diarrea osmótica de una secretora? Leucos en heces K+ en heces Brecha aniónica A y B son correctas La brecha aniónica es de utilidad para diferenciarlas, siendo el punto de corte de 125.

CASO CLÍNICO 2 Hombre de 55 años de edad con consumo frecuente de alcohol desde los 20 años. Hace 10 meses acudió a valoración de especialista porque presentaba diarrea crónica y pérdida de peso. Los estudios de extensión demostraron insuficiencia pancreática por pancreatitis crónica. ¿Cuál sería el mecanismo fisiopatológico de esta diarrea? Disminución en la superficie de absorción Mala digestión intraluminal Inactivación de enzimas Hipomotilidad intestinal La insuficiencia pancreática produce diarrea con datos de malabsorción por una mal digestión secundaria a una deficiencia de enzimas pancreáticas.

¿Cómo esperaría encontrar los carotenos séricos en este tipo de diarrea? Bajos Elevados Normales Elevados 2 veces de lo normal Carotenos bajos indican malabsorción. Pero no distingue entre las causas que afecta la pared intestinal de las de origen pancreático.

Si se le hiciera una prueba de D-xylosa, ¿qué resultado esperaría encontrar? D-xylosa 4 gr D xilosa distingue entre las causas que afectan la pared intestinal de las de origen pancreático. Si es menor a 5 gramos indica afección de la pared intestinal. Si es mayor a 5 sugiere mala digestión.

El tratamiento de elección en este paciente es: Loperamida Enzimas pancreáticas Subsalicilato de bismuto Colestiramina Tratamiento es suplementar al paciente con enzimas pancreáticas en cada alimento.

¿Qué porcentaje de pérdida de función pancreática se requiere para que un paciente tenga insuficiencia pancreática exocrina? 30% 60% 80% 90%

CASO CLÍNICO 3 Mujer de 67 años, tiene hipertensión arterial sistémica bajo tratamiento con amlodipino. Desde hace 6 meses presenta dolor abdominal epigástrico, plenitud temprana, edema de miembros inferiores. Sus laboratorios: Hb 13 g/dL, Cr 1 mg/dL, albúmina 2 g/dL, EGO sin proteinuria. Se le realizó una panendoscopia en la que se observó engrosamiento de pliegues gástricos. ¿Cuál es el diagnóstico más probable? Linfoma gástrico Síndrome de Zollinger-Ellison Gastroenteritis eosinofílica Enfermedad de Ménétrier El engrosamiento de pliegues gástricos aunado a la hipoalbuminemia de origen no renal sugiere una enteropatía perdedora de proteínas.

Con respecto a la hipoalbuminemia de esta paciente, es cierto que: Es por pérdidas renales Es secundaria a una enteropatía perdedora de proteínas No tiene relación con la sospecha clínica Amerita recolección de orina de 24 hr Tiene EGO sin proteinuria y los hallazgos endoscópicos correlacionan con una probable Enfermedad de Ménétrier.

¿Cuál de los siguientes es un diagnóstico diferencial? Cáncer gástrico Síndrome de Zollinger-Ellison Gastropatía hipertrófica asociada a H. pylori Todas las anteriores Todas las anteriores dan engrosamiento de pliegues gástricos similar al Ménétrier.

El diagnóstico definitivo de esta patología es por: Hallazgos endoscópicos Biopsia de pared gástrica Por la hipoalbuminemia Diagnóstico de exclusión Tratamiento de esta patología es: Gastrectomía Vigilancia endoscópica Inhibidor de la bomba de protones No amerita tratamiento Tratamiento con gastrectomía por riesgo asociado de cáncer gástrico. Sin embargo se puede optar por vigilancia endoscópica regular.

CASO CLÍNICO 4 Paciente masculino de 34 años, con antecedente de úlcera péptica asociada a H. pylori documentada hace

2 meses, recibió tratamiento de erradicación. Se le tomaron biopsias gástricas que reportaron metaplasia intestinal incompleta en cuerpo y antro sin displasia. Actualmente se encuentra asintomático. ¿Cuál de las siguientes es una indicación de erradicación de Helicobacter pylori? Antecedente de úlcera péptica Metaplasia intestinal incompleta Linfoma gástrico Todas las anteriores

Usted quiere saber si se logró la erradicación de H. pylori. ¿Qué estudio solicitaría? Serología para Helicobacter pylori Prueba de aliento No es necesario corroborar erradicación ya que la tasa de respuesta es >90% con terapia triple Biopsia La tasa de respuesta de erradicación de H. pylori es variable de acuerdo con el esquema elegido y las resistencias antimicrobianas de cada población.

¿Cuál sería el riesgo de metaplasia intestinal incompleta a largo plazo? Úlcera péptica Adenocarcinoma gástrico Perforación gástrica No existe ningún riesgo Metaplasia intestinal incompleta con sulfocianina es el de mayor riesgo de cáncer gástrico.

¿Qué tipo de metaplasia es la que tiene mayor riesgo de cáncer gástrico? Metaplasia intestinal completa Metaplasia intestinal incompleta Las dos presentan el mismo riesgo La metaplasia con sialomucinas + En el seguimiento de este paciente será necesario: Amerita seguimiento si se corrobora erradicación de H. pylori El pepsinógeno sirve como marcador tumoral Realizar panendoscopia de vigilancia No hacer nada Tiene metaplasia intestinal incompleta y extensa (afecta dos áreas del estómago) por lo que amerita vigilancia por riesgo incrementado de adenocarcinoma.

CASO CLÍNICO 5 Hombre de 56 años de edad acude por melena de 3 días de evolución. Se encuentra con TA 110/70mmHg, FC 110 lpm, FR 18 rpm, sin datos relevantes a la exploración física.

¿Cuál es el diagnóstico de este paciente? Sangrado de origen desconocido Sangrado de origen oculto Sangrado de tubo digestivo alto Sangrado de tubo digestivo bajo

La gastrina es producida por: Las glándulas oxínticas en estómago. Células parietales Células G Células principales

La melena sugiere sangrado de tubo digestivo alto. No es de origen oscuro ni desconocido porque tiene sangrado manifiesto que no ha sido abordado. El sangrado de tubo digestivo bajo se manifiesta con hematoquezia o rectorragia.

La gastrina se produce en el antro por células G. Las glándulas oxínticas contienen células principales que secretan pepsinógeno y las parietales que secretan FI y HCl.

La etiología más probable es: Úlcera esofágica Mallory-Weiss Esofagitis Úlcera péptica El estudio de elección en este caso es: Colonoscopia Panendoscopia Angiografía Tomografía de abdomen El agente causal más probable es: Cirrosis hepática Infección por H. pylori Consumo frecuente de AINES Gastritis biliar H. pylori es la principal causa de ulcera péptica y en segundo lugar están los AINES.

En la panendoscopia se observó una úlcera en antro recubierta de fibrina. ¿Qué tratamiento requiere? Tratamiento hemostático por vía endoscópica Sucralfato Ranitidina Erradicación de H. pylori Por ser Forrest III no amerita tratamiento endoscópico. Suponiendo que la causa sea H. pylori habría que solicitar prueba de aliento y si sale positiva dar tratamiento de erradicación, IBP durante 8 semanas.

Preguntas independientes Hormona que estimula principalmente la secreción de enzimas pancreáticas por las células acinares: Gastrina Secretina Colecistocinina Somatostatina Colecistocinina es el principal estimula para secreción de enzimas pancreáticas y la secretina para la secreción de líquido rico en cloro y bicarbonato.

¿Cuál de las siguientes enzimas ese encarga de hidrolizar almidones? Amilasa Carboxipeptidasas Nucleasas Tripsina La única que hidroliza almidones es la amilasa que es una enzima glucolítica. El resto son proteolíticas.

La distensión gástrica condiciona relajación del esfínter ileocecal. ¿Cómo se le llama a este reflejo? Reflejo enterogástrico Reflejo gastrocólico Reflejo gastroileal Reflejo entérico Una mujer de 54 años de edad refiere deseo de evacuar 30 min después del desayuno. ¿Cómo se le llama al siguiente reflejo? Reflejo enterogástrico Reflejo gastrocólico Reflejo intestino-intestinal Reflejo recto esfintérico Malabsorción intestinal / síndrome intestino irritable / Estreñimiento crónico / Isquemia mesentérica CASO CLÍNICO 1 Hombre de 60 años inició hace 3 meses con diarrea caracterizada por 5 evacuaciones diarias con esteatorrea. Refiere que la diarrea mejora con el ayuno y empeora con los alimentos. Además de la diarrea, ha tenido pérdida de peso de 3 kg desde el inicio de su padecimiento actual. Tiene historia de hemicolectomía derecha con anastomosis primaria a los 55 años por cáncer de colon. ¿Cómo clasificaría clínicamente la diarrea del paciente? Secretora Inflamatoria Malabsortiva Osmótica

Se acompaña de esteatorrea, pérdida de peso y mejora con el ayuno. Si bien la diarrea malabsortiva tiene un componente osmótico. En este caso el mecanismo fisiopatológico primario es malabsorción.

¿Cuál de las siguientes pruebas diagnósticas es de utilidad para diferenciar malabsorción originada en la pared intestinal vs malabsorción intraluminal? Prueba de aliento con lactulosa Elastasa pancreática fecal D-Xilosa Secretina en jugo pancreático La prueba inicial para diferenciar si la malabsorción tiene su origen en la pared intestinal o el lumen intestinal es la D-Xilosa. La D-Xilosa es un carbohidrato que se absorbe directamente al torrente sanguíneo en pacientes con pared intestinal intacta y se miden sus niveles en orina.

Se realizó prueba de D-Xilosa y fue reportada por debajo del valor de referencia. Asimismo se midieron los niveles de vitamina B12 que fueron reportados bajos y niveles de ácido fólico que fueron reportados por arriba del límite de referencia. Se interrogó al paciente y no toma ningún suplemento vitamínico. De acuerdo al cuadro clínico y antecedentes del paciente, ¿cuál es el mecanismo fisiopatológico más probable de la diarrea?

Pancreatitis crónica obstructiva Tumor en la cabeza del páncreas que obstruye el conducto pancreático Sobrepoblación bacteriana Enfermedad celíaca El paciente tiene historia de resección de colon derecho con pérdida de la válvula ileocecal. Esto ocasiona reflujo del contenido colónico al íleon terminal y por lo tanto sobrepoblación bacteriana. Por otro lado las alteraciones en los niveles vitamínicos son sugerentes de sobrepoblación bacteriana.

De acuerdo a su respuesta anterior, ¿cuál sería el mejor tratamiento a ofrecer en este caso? Cirugía de Whipple Dieta libre de gluten Ciclos de antibióticos Pancreatografía con colocación de endoprótesis El mejor tratamiento para sobrepoblación bacteriana es antibióticos cíclicos. Son de elección rifaximina, ciprofloxacino, amoxicilina con clavulanato, tetraciclinas, metronidazol. De preferencia deben alternarse los antibióticos para disminuir el riesgo de resistencia bacteriana.

¿Cuál es el mecanismo fisiopatológico de las alteraciones en los niveles vitamínicos? Anticuerpos anti células parietales/ingesta oculta de suplementos de folato Resección de íleon distal Consumo bacteriano de B12/producción bacteriana de ácido fólico

Carencia en la dieta de B12 e ingesta de suplementos de folato El mecanismo de las deficiencias vitamínicas en este caso es consumo de vitamina B12 por microbiota del intestino delgado y producción de ácido fólico por las mismas.

CASO CLÍNICO 2 Mujer de 45 años de edad con antecedente de estreñimiento crónico caracterizado por heces duras y en escíbalos (Bristol 1-2), acompañadas de pujo y tenesmo. Evacua diariamente y niega dolor abdominal. Refiere que tiene dicho padecimiento desde los 18 años de edad y utiliza fibra con respuesta parcial. Es sana y no tiene otros antecedentes de importancia. Sus exámenes de laboratorio son normales. ¿Qué prueba descartarías en el abordaje inicial de un paciente con constipación? Manometría anorrectal Pruebas de función tiroidea Calcio sérico Tacto rectal La manometría anorrectal no es un estudio en el abordaje inicial de la constipación. Se realiza después de haber descartado razonablemente causas secundarias ya sea clínicamente o con exámenes auxiliares.

Se le administró prueba terapéutica con Psyllium Plantago con respuesta parcial. ¿Cuál de los siguientes tratamientos es el más apropiado a continuación? Enemas de lactosa Biorretroalimentación Polietilenglicol Ningún tratamiento hasta no realizar colonoscopia El tratamiento inicial de la constipación consiste en fibra soluble y laxantes osmóticos. El polietilenglicol es un laxante osmótico. La lactosa también es un laxante osmótico sin embargo la incomodidad de administrar enemas supera al beneficio.

Se inició el tratamiento recomendado por usted sin mejoría adicional. La paciente refiere que adicionalmente tiene episodios de dolor durante la defecación y a veces requiere introducir su dedo para extraer materia fecal residual y pujo excesivo con tenesmo. ¿Cuál de las siguientes pruebas diagnósticas se encuentra indicada a continuación?

Colonoscopia Enema baritado Manometría anorrectal Tránsito colónico La paciente tiene síntomas sugerentes de disinergia defecatoria u obstrucción funcional. En estos casos el abordaje de elección es manometría anorrectal.

¿Cuál es el mecanismo fisiopatológico más probable de los síntomas de la paciente? Obstrucción anorrectal por tumor Contracción paradójica del puborrectal Tránsito colónico lento con inercia colónica Mal apego al tratamiento Uno de los mecanismos de disinergia defecatoria es contracción paradójica del puborrectal.

¿Cuál es el tratamiento de elección para la condición de la paciente? Biorretroalimentación Agregar procinético Enemas periódicos Resección anterior baja de tumor colónico El tratamiento de elección de los trastornos funcionales defecatorios es la biorretroalimentación.

CASO CLÍNICO 3 Hombre de 32 años con dolor abdominal tipo cólico intermitente desde hace 2 años, asociado a episodios de constipación. El dolor se presenta 1 vez por semana y mejora con las evacuaciones. Los períodos sin síntomas sus evacuaciones son normales, niega otros síntomas y sus exámenes de laboratorio son normales. ¿Cuál es el diagnóstico más probable en este caso? Constipación funcional Síndrome de intestino irritable Cáncer de recto Disinergia defecatoria El paciente cumple con criterios de Roma para SII. Hay que recordar que es necesario el dolor abdominal para integrar el diagnóstico de SII.

De acuerdo a su diagnóstico, ¿cuál es una indicación de realizar colonoscopia en pacientes con este padecimiento? Todos los pacientes con constipación Todos los pacientes con SII Edad mayor a 50 años Tenesmo durante las evacuaciones El síndrome de intestino irritable es un diagnóstico clínico y en ausencia de síntomas de alarma puede diagnosticarse razonablemente sin exámenes auxiliares. La excepción es en pacientes con diarrea (Pruebas de función tiroidea y anticuerpos anti-transglutaminasa) y en pacientes con datos de alarma o mayores de 50 años (colonoscopia).

¿Cuál opción considera apropiada como primera opción para el tratamiento del dolor abdominal en este caso? Fibra soluble Fibra no soluble

Antiespasmódicos Antidepresivos tricíclicos El tratamiento del síndrome de intestino irritable es de acuerdo al síntoma. El tratamiento inicial de elección para el dolor abdominal son los antiespasmódicos.

Pasados 4 meses, se agregó al cuadro del paciente, además del dolor abdominal, episodios de distensión abdominal y hábito defecatorio alternante con diarrea (25%) / estreñimiento (30%). Se realizaron exámenes microbiológicos de materia fecal sin encontrar alteraciones. No hay cambios en otros síntomas y los laboratorios generales son normales. ¿Cuál de las siguientes pruebas diagnósticas considera apropiada para solicitar a continuación? Colonoscopia Anticuerpos anti-transglutaminasa Sangre oculta en heces Tránsito intestinal Una forma de manifestación de enfermedad celíaca es como simulador de síndrome de intestino irritable variedad diarrea por lo tanto, en este caso está indicado realizar tamizaje con AC anti TG IgA.

CASO CLÍNICO 4 Mujer de 80 años, obesa, diabética e hipertensa, con historia de infarto agudo al miocardio hace 10 años tratado con angioplastía con colocación de stent medicado. Inició hace 6 meses con dolor abdominal intermitente postprandial (1 hora después de comer). Dicho dolor es intenso y la paciente disminuyó su ingesta por miedo al dolor. Durante este período ha bajado 8 kg de peso. ¿Cuál es el mecanismo subyacente más probable de los síntomas de la paciente? Trombosis venosa mesentérica Trombosis arterial mesentérica Placa ateromatosa arterial mesentérica Obstrucción intestinal intermitente Es un caso típico de angina intestinal o isquemia mesentérica crónica (Dolor postprandial y sitofobia). La angina intestinal es secundaria a disminución predecible del flujo sanguíneo por obstrucción fija (placa ateromatosa) en la circulación arterial mesentérica. En este caso la paciente adicionalmente tiene múltiples factores de riesgo de enfermedad cardiovascular que usualmente se encuentran presentes en estos casos.

Dos días después, la paciente acudió al servicio de urgencias por dolor abdominal de 3 horas de evolución sin relación con los alimentos. La exploración física únicamente con ruidos intestinales discretamente disminuidos en intensidad pero la paciente tiene un dolor desproporcionado a los hallazgos.

¿Cuál es el mecanismo más probable en este caso del cuadro agudo?

es la causa más probable del cambio en el patrón de los síntomas de la paciente?

Embolismo arterial mesentérico Trombosis arterial mesentérica Placa ateromatosa que incrementó su tamaño Íleo por congestión capilar del intestino delgado

Nueva trombosis arterial mesentérica Trombosis venosa mesentérica Isquemia colónica Enfermedad diverticular

Si bien el mecanismo más frecuente de isquemia mesentérica aguda es embolismo arterial. En este caso existe un factor de riesgo claro para trombosis in situ (presencia de placa ateromatosa).

La paciente tuvo un episodio de deshidratación con inestabilidad hemodinámica que muy probablemente condicionó hipoperfusión a las zonas penumbra del colon (Sudeck y Griffith). La isquemia colónica es el síndrome más frecuente de isquemia mesentérica.

¿Cuál es el siguiente estudio recomendado en este caso? Angiotomografía de abdomen Resonancia magnética de abdomen Angiografía mesentérica urgente Angiografía mesentérica electiva El estudio inicial en un paciente con isquemia mesentérica sin datos de irritación peritoneal y con menos de 12 horas de evolución de los síntomas es angiotomografía de abdomen. Es útil para planear el tratamiento ulterior. En caso de encontrar datos de perforación, neumatosis o gas portal se requeriría tratamiento quirúrgico. En caso de no encontrar datos sugerentes de perforación o de intestino no viable, el siguiente paso es angiografía.

El estudio mencionado, mostró obstrucción total al flujo sanguíneo cerca del ostium de la arteria mesentérica superior. No hay aire libre subdiafragmático ni datos de neumatosis intestinal/portal. Las condiciones de la paciente no han cambiado desde su llegada (hace 1 hora) ¿Cuál es el siguiente paso en el tratamiento de la paciente?

Anticoagulación y vigilancia Anticoagulación y angiografía Laparotomía exploratoria Antiagregantes y vigilancia El estudio auxiliar (TAC), mostró obstrucción probablemente secundaria a trombosis in situ de placa de ateroma. De acuerdo a los hallazgos, ausencia de irritación peritoneal y datos de perforación se sugiere angiografía diagnóstica/terapéutica (trombectomía, angioplastía e infusión de papaverina).

El tratamiento recomendado fue exitoso y la paciente fue egresada a la semana de su ingreso y se ha mantenido asintomática en el seguimiento. Un año después, acude nuevamente a urgencias por diarrea acuosa abundante (10 evacuaciones diarias) después de viaje. En la exploración física se evidencia ortostatismo y sequedad de mucosas. Los exámenes de laboratorio destacan por Na 128 mEq/L, Cr 2.5 (previa de 1.2), se realizaron coprocultivos y coproparasitoscópicos que fueron negativos. Se realizó hidratación intravenosa con mejoría, sin embargo 2 días después, inició con dolor abdominal tipo cólico súbito en mesogastrio, acompañado de una evacuación hematoquézica en moderada cantidad. Los síntomas duraron 1 día y se autolimitaron con mejoría ulterior. ¿Cuál

CASO CLÍNICO 5 Hombre de 35 años de edad. Inició hace 8 meses con dolor abdominal intermitente, tipo cólico, acompañado de evacuaciones duras con pujo y tenesmo. El dolor mejora con la defecación. Niega otros síntomas y durante los períodos libres de dolor abdominal su hábito intestinal es normal. ¿Cuál de los siguientes es el diagnóstico diferencial más apropiado en este caso? Constipación crónica funcional con disinergia defecatoria Constipación crónica funcional de tránsito colónico lento Síndrome de intestino irritable – constipación Constipación crónica funcional de tránsito colónico normal La presencia de dolor abdominal que se relaciona con la defecación y con el cambio en el hábito intestinal es altamente sugerente de síndrome de intestino irritable. Este paciente no tiene datos de alarma que obliguen a realizar un abordaje paraclínico extenso.

¿Cuál de las siguientes características clínicas es indispensable para integrar el diagnóstico previo? Pujo Tenesmo Dolor abdominal Constipación La presencia de dolor abdominal es fundamental para poder integrar el diagnóstico de síndrome de intestino irritable.

30 años después, inició con constipación de reciente aparición caracterizada por evacuaciones acintadas y disminuidas en frecuencia con pujo y tenesmo, se mantuvo estable con tratamiento sintomático durante esos años. Al interrogatorio refiere que el cuadro apareció desde hace 2 meses y ha empeorado. No siempre se acompaña de dolor abdominal. ¿Cuál de los siguientes diagnósticos es el menos probable dentro del diagnóstico diferencial en este caso?

Constipación funcional Carcinoma colorrectal Hipercalcemia Hipotiroidismo La forma de presentación sugiere una constipación secundaria. Todas las presentadas son causas secundarias, excepto constipación funcional.

Este examen complementario se puede evitar realizarse de manera inicial: Colonoscopia Pruebas de función tiroidea Electrolitos séricos Manometría anorrectal Antes de realizar abordaje para constipación funcional y trastornos defecatorios en un paciente >50 años con cambio reciente en el hábito intestinal, se sugiere descartar causas secundarias.

¿En cuál de los siguientes escenarios clínicos solicitaría una colonoscopia desde el inicio del abordaje? Mujer de 40 años con constipación de 20 años de evolución Mujer de 45 años con pujo y tenesmo de 15 años de evolución Mujer de 30 años con constipación acompañada de dolor abdominal intermitente desde hace 5 años que mejora al evacuar Mujer de 40 años con constipación de reciente aparición acompañada de síndrome anémico sin sangrado manifiesto La presencia de datos de alarma (anemia, pérdida de peso, cambios recientes en el hábito intestinal, antecedentes familiares de cáncer de colon, sangrado) son sugerentes de causas secundarias de constipación y requieren mayor abordaje.

CASO CLÍNICO 6 Hombre de 50 años, hospitalizado por Infarto agudo al miocardio. Como manifestación inicial tuvo choque cardiogénico. Recibió terapia de reperfusión mediante cateterismo con colocación de stent medicado en arteria descendente anterior. Ha evolucionado favorablemente, egresó de la unidad coronaria a piso y se encuentra esperando su alta hospitalaria, sin embargo inició hace 2 horas con dolor abdominal tipo cólico seguido por 2 evacuaciones hematoquézicas. A la exploración: signos vitales normales, no hay datos de irritación peritoneal. El paciente se negó a recibir abordaje diagnóstico y continuó únicamente con el tratamiento ya establecido a base de anticoagulación y antiagregantes por el síndrome coronario agudo. Su evolución fue favorable y fue egresado a las 48 horas.

¿Cuál es la causa más probable de los síntomas gastrointestinales en este paciente? Isquemia mesentérica aguda arterial por embolismo Isquemia mesentérica crónica por placa ateromatosa Isquemia / reperfusión colónica Isquemia mesentérica por trombosis venosa Se trata de un cuadro típico de colitis isquémica. La causa más frecuente de isquemia intestinal es la isquemia colónica y el 95% de los casos se debe a cuadros no oclusivos de la circulación. En este caso existen dos factores claros que pudieron contribuir (choque cardiogénico y procedimiento endovascular arterial).

Este hallazgo no corresponde a alguna de las pruebas diagnósticas en este caso: Obstrucción total del flujo de la arteria mesentérica superior Mucosa edematosa, frágil con úlceras longitudinales y áreas de hemorragia. Signo de la huella digital en TAC Engrosamiento de colon izquierdo La localización más frecuente de la isquemia colónica es el colon izquierdo. El estudio inicial de elección es la TAC y se sugiere realizar colonoscopia temprana para corroborar el diagnóstico. Recordando esto y que la causa más frecuente de isquemia colónica es no oclusiva, es poco probable en un paciente de estas características y con evolución favorable tenga datos por imagen de trombosis arterial mesentérica.

Esta área del intestino puede tener susceptibilidad a la isquemia: Unión rectosigmoidea (Punto de Sudeck) Ángulo esplénico (punto de Griffith) Recto Yeyuno

menos

El recto al tener doble circulación (AMS y AMI) es poco probable que sufra eventos de isquemia.

Puedes diferir la cirugía, si se presenta este signo: Neumatosis intestinal Neumatosis portal Necrosis colónica en colonoscopia Hematoquezia moderada Sólo la hematoquezia masiva es una indicación de cirugía

¿Cuál es la forma más común de comportamiento de esta entidad? Evolución favorable y resolución Deterioro y perforación intestinal Mejoría inicial con deterioro ulterior Progresión rápida con sepsis y choque séptico La mayoría de los casos de colitis isquémica tienen un curso benigno y autolimitado. Incluso muchos casos no se diagnostican debido a que el paciente no acude a consulta. Los casos con peor pronóstico son los casos con isquemia del colon derecho, SRIS, leucocitosis o neumatosis en los estudios de imagen.

Oclusión intestinal/Enfermedad Inflamatoria Intestinal/Hepatopatía por alcohol CASO CLÍNICO 1 Mujer previamente sana de 70 años de edad, inició hace 5 horas con dolor abdominal tipo cólico intenso. Posteriormente se agregó al cuadro vómito abundante de contenido gastrobiliar, distensión abdominal e imposibilidad para evacuar. A la exploración destaca la presencia de ruidos metálicos, sin datos de irritación peritoneal. Tiene antecedentes quirúrgicos de colecistectomía, apendicectomía e histerectomía Además de los exámenes generales, ¿cuál es el primer estudio a solicitar en este caso? Radiografía de abdomen Tomografía de abdomen Ultrasonido abdominal Serie esofagogastroduodenal En un cuadro típico de oclusión intestinal el primer estudio a solicitar es una radiografía de abdomen. En caso de tener dudas respecto al diagnóstico o bien clarificar y buscar causas, el siguiente estudio es una tomografía de abdomen.

¿Cuál de los siguientes hallazgos es el más probable en el estudio solicitado? Niveles hidroaéreos sin gas en el colon Niveles hidroaéreos con gas en el colon Líquido libre intraabdominal Aire libre subdiafragmático Lo más probable es que se trate de una oclusión intestinal mecánica. El hallazgo típico por radiografía es presencia de niveles hidroaéreos sin gas en el colon.

¿Cuál es la etiología más probable del cuadro de la paciente? Hipocalemia Isquemia mesentérica Adherencias Medicamentos La paciente tiene historia de tres cirugías abdominales. Además, la causa más frecuente de obstrucción intestinal mecánica es adherencias por cirugías abdominales previas.

¿Cuál no es una opción terapéutica apropiada en este momento? Laparotomía exploradora Hidratación Colocación de sonda nasogástrica Analgesia

A menos de que haya datos de irritación peritoneal o hallazgos en la imagen que sugieran perforación, el manejo inicial de una oclusión intestinal es conservador.

La paciente mejoró después de 24 horas con el manejo indicado. ¿Cuál es el pronóstico en este caso? La tasa de recurrencia es nula Se espera recurrencia de aproximadamente 20% La tasa de recurrencia es de aproximadamente 60% La tasa de recurrencia es de 5% La mayoría de los estudios de recurrencia de oclusión intestinal refieren una tasa de aproximadamente 20% en los siguientes 5 a 10 años del evento.

CASO CLÍNICO 2 Mujer de 29 años de edad con bulimia sin antecedentes de importancia ni otras comorbilidades. Tiene historia de uso de múltiples métodos “purgativos”. Inició hace 6 horas con vómito de contenido gastrobiliar, distensión abdominal e imposibilidad para evacuar. En la exploración física se encuentra ausencia de ruidos intestinales. No hay datos de irritación peritoneal. ¿Cuál es el mecanismo fisiopatológico más probable en este caso? Mecánico Metabólico Psicógeno Isquémico Es un cuadro típico de obstrucción intestinal motora (íleo paralítico). Después del íleo postquirúrgico donde el antecedente de cirugía es evidente, las causas metabólicas son las más frecuentes. La isquemia también puede ocasionar íleo sin embargo debido a los antecedentes de la paciente y el cuadro clínico, la causa más probable es metabólica (hipocalemia).

Usted solicita radiografía de abdomen, ¿cuál de los siguientes hallazgos espera encontrar con mayor probabilidad? Dilatación difusa de asas con gas colónico Dilatación difusa de asas sin gas colónico Dilatación difusa de asas con zona de transición clara Aire libre subdiafragmático El patrón radiológico típico de la obstrucción intestinal motora (íleo paralítico) es dilatación difusa de asas con niveles hidroaéreos con presencia de gas en el colon. La ausencia de gas y la presencia de zona de transición son sugerentes de oclusión mecánica.

Además del cuadro clínico principal, la paciente tiene dolor intenso del tobillo por esguince a la hora de bajarse del coche hacia el hospital. ¿Cuál de los

siguientes tratamientos estaría contraindicado en este caso? Colocación de sonda nasogástrica Hidratación y reposición de electrolitos Analgesia con AINES Analgesia con opiáceos En caso de requerir analgesia por cualquier motivo, el uso de opiáceos no es deseable debido a que contribuirían a perpetuar la oclusión intestinal motora.

Después de tranquilizar a la paciente, se procede con un interrogatorio minucioso, ¿cuáles de los siguientes métodos purgativos utilizados por la paciente, es más probable que haya contribuido al cuadro?

Vómito y uso de diuréticos Ejercicio anaeróbico y uso de laxantes estimulantes Atracón reciente y consumo de fentermina Uso de laxantes estimulantes y atracones El vómito y el uso de diuréticos son factores que contribuyen al desarrollo de hipocalemia que es una etiología clara de íleo paralítico.

¿Cuál de las siguientes alteraciones de laboratorio, espera encontrar con mayor probabilidad en este caso? Acidosis metabólica e hipercalemia Alcalosis metabólica e hipocalemia Alcalosis respiratoria e hipocalemia Alcalosis metabólica e hipercalcemia Debido a las causas subyacentes en este caso del íleo paralítico, lo más frecuente en un paciente con vómito y/o consumo de diuréticos, es la presencia de alcalosis metabólica hipocalémica.

CASO CLÍNICO 3 Hombre de 25 años de edad sin antecedentes de importancia. Inició hace 1 semana con evacuaciones sanguinolentas intermitentes en escasa cantidad, acompañadas de pujo y tenesmo por lo que acude a consulta. A la exploración física, paciente se encuentra con signos vitales normales, no tiene dolor abdominal a la palpación y no hay alteraciones al tacto rectal, aunque en el guante se observan restos hemáticos. ¿Cuál de las siguientes no es una conducta apropiada en este momento? Realizar coproparasitoscópico y coprocultivo Toxina de C. difficile Realizar colonoscopia Medición de reactantes de fase aguda

La causa más frecuente de síndrome disentérico son las infecciones. En el abordaje inicial en un paciente joven con síndrome disentérico sin otros datos de alarma no está indicado de forma inicial la realización de colonoscopia. Dicho estudio debe realizarse una vez que no se encuentran causas infecciosas como abordaje de sangrado de tubo digestivo bajo.

En el abordaje no se encontraron causas infecciosas. El paciente decidió irse de alta voluntaria y se automedicó con antibióticos. Regresó 6 meses después por persistencia de los síntomas. Usted decide realizar colonoscopia. ¿Qué hallazgos esperaría usted encontrar?

Eritema y pérdida del patrón vascular que respetan el recto Úlceras en botón de camisa Eritema y pérdida del patrón vascular que se extiende de distal a proximal Úlceras serpiginosas alternando con áreas de mucosa sana El cuadro clínico es altamente sugestivo de Colitis ulcerativa crónica inespecífica (CUCI: hematoquezia con pujo y tenesmo). Dicha enfermedad se caracteriza por afección universal del recto, continua, circunferencial y de distal a proximal..

La biopsia reportó colitis crónica con plasmocitosis basal con ramificación de criptas. ¿Cuál de las siguientes no es una característica típica de esta enfermedad? Afección únicamente del colon Ileítis retrógrada en algunos casos Enteritis segmentaria en cualquier sitio del tubo digestivo Inflamación circunferencial continúa en recto La CUCI se caracteriza por afección circunferencial continua y simétrica del recto que se extiende de distal a proximal. Dicha entidad afecta únicamente el colon y en aproximadamente 20% de los casos de colitis extensa (pancolitis) puede observarse ileítis retrógrada. La enteritis segmentaria es una característica de la enfermedad de Crohn.

¿Cuál de los siguientes factores es poco probable que haya contribuido al cuadro? Suspensión de tabaquismo de larga evolución Tabaquismo de larga evolución Uso de AINES Raza caucásica El tabaquismo se considera un factor protector para el desarrollo de CUCI. Sólo 13% de los pacientes con CUCI son fumadores. La mayoría son no fumadores o fumadores previos. Existe un gradiente de norte a sur para enfermedad inflamatoria intestinal con mayor prevalencia en los países con razas caucásicas.

Este fármaco es inapropiado en el tratamiento inicial de esta enfermedad: Prednisona Azatioprina Mesalazina Sulfasalazina Las tiopurinas (azatioprina y mercaptopurina) no se consideran fármacos para inducción de la remisión en CUCI debido a que el tiempo de inicio de acción es muy prolongado (aprox. 6 semanas). Los esteroides y los aminosalicilatos son medicamentos pilares para la inducción a la remisión en este grupo de pacientes.

CASO CLÍNICO 4 Mujer de 24 años de edad. Inició hace 2 años con episodios de dolor abdominal localizado en fosa ilíaca derecha. Ha acudido a urgencias en dos ocasiones ante la posibilidad de apendicitis sin embargo dicho diagnóstico se ha descartado. En uno de los estudios de imagen se observó engrosamiento en los últimos 30 cm del íleon terminal. En esta ocasión acudió al presentar 2 evacuaciones hematoquézicas por lo que se realizó colonoscopia encontrando úlceras lineales profundas en íleon terminal y en colon derecho con áreas de mucosa sana circundante. ¿Cuál de las siguientes aseveraciones no corresponde a esta enfermedad? Afección transmural Afección exclusiva del colon e íleon distal Afección de cualquier sitio del tubo digestivo Formación de fístulas enterocutáneas La enfermedad de Crohn puede afectar cualquier sitio del tubo digestivo y su afección es transmural. Existen los fenotipos no perforante no estenosante, estenosante y penetrante. Este último se asocia a la formación de fístulas.

¿Cuál de los siguientes hallazgos histopatológicos es característico de esta enfermedad? Plasmocitosis basal con ramificación de criptas Granulomas caseificantes Granulomas no caseificantes Células inflamatorias con núcleos gigantes y cuerpos de inclusión La presencia de granulomas no caseificantes es altamente sugestiva de enfermedad de Crohn, lamentablemente su rendimiento diagnóstico es bajo ya que solo se logran encontrar en 5% de las biopsias mucosas y en 30% de los especímenes de resección intestinal.

¿Cuál de las siguientes manifestaciones extraintestinales no se encuentra asociada a esta entidad? Uveítis Sacroileítis Eritema nodoso Dermatitis herpetiforme La dermatitis herpetiforme se encuentra asociada a enfermedad celíaca no a Enfermedad de Crohn.

¿Cuál de los siguientes es el sitio afectado más frecuente en esta enfermedad? Recto Íleon distal y colon derecho Estómago y duodeno Yeyuno La localización más frecuente de la enfermedad de Crohn es la ileocolónica.

¿Cuál medicamento descartarías para el manejo de esta enfermedad? Mofetil micofenolato Prednisona Budesonida Infliximab El mofetil micofenolato no es un medicamento utilizado en el manejo de pacientes con esta enfermedad.

CASO CLÍNICO 5 Hombre de 55 años acude al servicio de urgencias debido a ictericia, fiebre y dolor en hipocondrio derecho. A la exploración se encuentra taquicárdico. La exploración de abdomen muestra hepatomegalia y dolor a la palpación del hipocondrio derecho. Tiene antecedente de uso de valproato por crisis convulsivas y niega ingesta de alcohol. En los exámenes de laboratorio, destaca hiperbilirrubinemia de 15 mg/dL, BD 7 mg/dL, BI 8 mg/dL, FA 130 U/L, GGT 400 U/L. No hay reactivo para medición de aminotransferasas. BH con Hb de 11 g/dL con VGM de 110, leucocitos de 25000 / µL, INR 2.6. Creatinina 0.8 mg/dL. Se realizó ultrasonido de hígado y vías biliares observando hígado brillante sin dilatación de la vía biliar y la vesícula biliar sin litos.

¿Qué diagnóstico descartarías por no corresponder con el cuadro? Hepatitis viral Lesión hepática inducida por fármacos Hepatitis alcohólica Adenocarcinoma de páncreas

¿Cuál de las siguientes terapéuticas te parece la menos viable en este caso? Prednisona Abstinencia Pentoxifilina Trasplante hepático

El cuadro clínico es sugestivo de inflamación del parénquima hepático. En este momento dentro del diagnóstico diferencial debemos tomar en cuenta cualquier causa de hepatitis aguda (virus, toxinas, alcohol). No hay dilatación de la vía biliar y el patrón de las PFH no sugiere en absoluto causas obstructivas (cáncer de páncreas).

Para considerar a un paciente candidato a trasplante hepático es necesario contar con abstinencia de por lo menos 6 meses. Los esteroides son el tratamiento de primera línea en hepatitis alcohólica y la pentoxifilina puede utilizarse en pacientes con falla renal, infecciones, sangrado o donde el uso de esteroide se encuentra contraindicado.

Se realizó biopsia hepática transyugular, observando esteatosis macrovesicular con infiltrado neutrofílico. Con esta información y el cuadro clínico, ¿cuál sería su primer diagnóstico diferencial? Lesión hepática por valproato Esteatohepatitis no alcohólica Esteatohepatitis alcohólica Colangitis ascendente Muchos pacientes niegan la ingesta de alcohol inicialmente. La esteatohepatitis alcohólica es difícil de distinguir histológicamente de la no alcohólica sin embargo la presentación de la segunda, no es aguda con hiperbilirrubinemia, fiebre y leucocitosis. Además existen varias pistas que orientan a consumo de alcohol (elevación de la GGT, macrocitosis en la BH).

¿Cuál es la fisiopatología subyacente más probable en este caso? Toxicidad por acetaldehído y estrés oxidativo Esteatosis macrovesicular por síndrome metabólico Hipersensibilidad al valproato Obstrucción del colédoco con infección bacteriana secundaria El principal mecanismo de toxicidad del alcohol es toxicidad por acetaldehído. La producción de radicales libres mediante el sistema microsomal produce daño y estrés oxidativo.

El laboratorio analizó la muestra inicial congelada para completar las pruebas de función hepática. ¿Cuál de los siguientes patrones de aminotransferasas es probable encontrar con mayor probabilidad en este caso? ALT 500 U/L, AST 450 U/L ALT 2500 U/L, AST 3000 U/L ALT 100 U/L, AST 220 U/L ALT 125 U/L, AST 100 U/L La hepatitis alcohólica de forma característica no eleva las aminotransferasas de forma masiva. Usualmente son elevaciones de leves a moderadas y el patrón característico es un predominio de la AST sobre la ALT (2:1).

CASO CLÍNICO 6 Hombre de 50 años con historia de consumo de alcohol excesivo. Acude a urgencias por incremento en el perímetro abdominal de 1 semana de evolución acompañado de edema de miembros inferiores. La presencia de este hallazgo descarta patología por consumo de alcohol excesivo: Hipertrofia parotídea Anemia macrocítica Anillo de Kayser-Fleischer Elevación de la GGT El anillo de Kayser-Fleischer se observa en la enfermedad de Wilson.

Al interrogatorio dirigido, el paciente refiere que tomaba aproximadamente 300 mL de ron de 38GL diariamente durante varios años. ¿Cuál es el consumo diario aproximado en gramos de alcohol de este paciente? 50 gramos 20 gramos 60 gramos 90 gramos Para calcular el consumo de alcohol en gramos se multiplica la cantidad de la bebida en mL por el porcentaje de alcohol, en este caso (0.38) ya que se trata de una bebida de 38 grados GL. El resultado es el consumo de alcohol en mL. Posteriormente para convertir mL a gramos se multiplica por 0.78 (1 mL de alcohol pesa 0.78 gramos). En este caso 300 mL x 0.38=114 mL de alcohol. 114 x 0.78=88.92 gramos.

¿Cuál es la ingesta máxima diaria recomendada de alcohol? 60 gramos de alcohol en hombres 30 gramos de alcohol en hombres 30 gramos de alcohol en mujeres 20 gramos de alcohol en hombres

La ingesta máxima recomendada diariamente es de 20 gramos en mujeres y 30 en hombres. A partir de 60 gramos el riesgo de esteatohepatitis alcohólica aumenta considerablemente.

¿Cuál es el tratamiento que se ha asociado a mejor pronóstico en estos casos? Trasplante hepático temprano Abstinencia Uso de disulfiram Baclofeno La abstinencia es el pilar del tratamiento en cirrosis alcohólica. Los demás tratamientos se consideran auxiliares, algunos para lograr abstinencia, otros para tratar complicaciones.

El paciente continuó bebiendo por 2 años más. Es llevado por sus familiares a urgencias por encefalopatía. Actualmente lleva 1 semana sin tomar. Tiene ictericia, encefalopatía grado 3 con rueda dentada y ascitis a tensión. Este paciente no es candidato idóneo para: Endoscopia USG hepático Paracentesis Evaluación de trasplante hepático Hepatitis no infecciosa / Cirrosis/ Tumores de hígado CASO CLÍNICO 1 Mujer de 23 años de edad previamente sana. Inició hace 3 meses con fatiga y artralgias. Posteriormente se agregaron al cuadro dolor en hipocondrio derecho por lo que acude a consulta. Exámenes de laboratorio: ALT 300 U/L, AST 280 U/L, FA 120 U/L, BT 1.8 mg/dL, BD 0.8 mg/dL, BI 1 mg/dL. Globulina de 6 g/dL. ¿Cuál de los siguientes auxiliares de diagnóstico evitarías en la evaluación inicial de esta paciente? Ultrasonido de hígado y vías biliares Colangiorresonancia magnética Serología para VHA Serología para VHE Se trata de una paciente con un cuadro clínico de hepatitis aguda. El patrón de las PFH es hepatocelular y no hay datos de colestasis por lo que no está justificado realizar colangiorresonancia. El abordaje inicial de la hepatitis aguda requiere descartar causas virales.

Los exámenes complementarios no mostraron alteraciones. Se solicitaron autoanticuerpos encontrando ANAs positivos (1:160), AML positivos (1:160). El hepatólogo sugirió realización de biopsia hepática. ¿Cuál de los siguientes hallazgos histológicos espera usted encontrar?

Destrucción biliar con lesión ductal florida Ductopenia Hepatitis lobulillar con polimorfonucleares Hepatitis de interfase con células plasmáticas De acuerdo al cuadro clínico y los hallazgos, parece que se trata de una hepatitis autoinmune. El hallazgo histológico típico de esta entidad es hepatitis de interfase. En algunos casos puede observarse lobulillar sobre todo en etapas tempranas pero el infiltrado no es neutrofílico y usualmente se observa más en pacientes con falla hepática aguda.

¿Cuál es la explicación de la elevación de las globulinas plasmáticas en esta paciente? Translocación bacteriana Elevación monoclonal de la IgG Elevación policlonal de la IgG Elevación policlonal de la IgM La hepatitis autoinmune característicamente se asocia a hiperglobulinemia IgG.

¿Cuál es el tratamiento inicial de elección para esta entidad? Esteroides y azatioprina Azatioprina Mofetil micofenolato Tacrolimus El tratamiento inicial de la hepatitis autoinmune puede ser con esteroide como monoterapia en dosis altas (60 mg por día de PDN) o bien la combinación de esteroide con azatioprina. Esta combinación ayuda a ahorrar dosis de esteroide y es la más utilizada en la actualidad. Los pacientes con cirrosis y colestasis tienen mayor riesgo de desarrollar toxicidad por tiopurinas por lo que en estos pacientes se prefiere sólo esteroide.

¿Cuáles son las metas terapéuticas y la duración necesaria del tratamiento en esta paciente? El tratamiento puede suspenderse al normalizar las transaminasas y las globulinas después de 6 meses El tratamiento puede suspenderse al normalizar las transaminasas y globulinas después de 18 meses La suspensión del tratamiento requiere nueva biopsia para confirmar remisión histológica y su duración es de 2 años El tratamiento debe continuar de por vida en todos los pacientes en dosis de mantenimiento al lograr normalización de transaminasas Para suspender el tratamiento se requiere una duración de 2 años y debe confirmarse remisión histológica antes de suspenderlo.

CASO CLÍNICO 2 Mujer de 44 años de edad con historia de hipotiroidismo en tratamiento actual con levotiroxina. Inició hace 3 meses con fatiga y prurito. El prurito es incapacitante. Su esposo refiere que los últimos meses

ha notado un cambio discreto en la coloración de la piel de la paciente y la salida de manchas amarillentas en los párpados. En los laboratorios destaca BT 1 mg/dL, FA 800 U/L, GGT 500 U/L, Albúmina 4 g/dL, Globulinas 5 g/dL. ¿Cuál de los siguientes hallazgos en los exámenes complementarios, esperaría usted encontrar con mayor probabilidad en esta paciente? Hiperglobulinemia a expensas de IgG Anticuerpos anti-músculo liso positivos Anticuerpos anti-mitocondriales positivos Colangiorresonancia con vía biliar arrosariada Se trata de un cuadro típico de colangitis biliar primaria. 95% de los pacientes con esta entidad, tienen anticuerpos anti-mitocondriales positivos.

En caso de realizar biopsia hepática, ¿cuál es el hallazgo esperable? Lesión ductal florida y ductopenia Inflamación colangiolar en capas de cebolla Hepatitis de interfase con células plasmáticas Hepatitis lobulillar con colestasis intrahepática Al ser un cuadro típico de colangitis biliar primaria, el hallazgo histológico típico es lesión ductal florida con granulomas y ductopenia. La inflamación colangiolar en capas de cebolla es de CEP y la hepatitis de interfase es de hepatitis autoinmune.

¿Cuál es el tratamiento de elección en esta entidad que se ha asociado a mejoría del pronóstico? Colestiramina Citalopram Prednisona Ácido ursodesoxicólico El ácido ursodesoxicólico es el tratamiento de elección en la CBP y los pacientes que responden bioquímicamente tienen un mejor pronóstico que aquellos que no mejoran.

¿Cuál de los siguientes tratamientos se considera como primera elección para el tratamiento del prurito? Citalopram Colestiramina Ácido ursodesoxicólico Plasmaféresis El tratamiento inicial para los pacientes con prurito es la colestiramina. La plasmaféresis se reserva para casos refractarios y en último lugar el trasplante hepático para prurito intratable.

¿Cuál es la explicación de la aparición de manchas amarillentas en los párpados? Acumulación de sales biliares Acumulación de colesterol Vasculitis Acumulación de glucosaminoglucanos

Las lesiones amarillentas son xantomas por hipercolesterolemia. Los pacientes con CBP tienen niveles elevados de colesterol y curiosamente no tienen un riesgo cardiovascular elevado.

CASO CLÍNICO 3 Hombre de 240 años de edad previamente sano. Tiene como antecedente un hermano fallecido a los 18 años de edad por “hepatitis aguda” y una hermana de 30 años con trastorno del movimiento no diagnosticado y cirrosis hepática. Inició hace 10 semanas con fatiga. Hace 2 semanas se agregó ictericia. Acudió con múltiples médicos quienes dieron tratamiento de soporte para “hepatitis viral aguda” sin mejoría. Hace 2 días inició con alteración del estado de alerta con somnolencia. Actualmente tiene encefalopatía grado 3. Los exámenes de laboratorio: BT 5 mg/dL, BD 1 mg/dL, BI 4 mg/dL, ALT 100 U/L, AST 220 U/L, FA 30 U/L. BH con Hb de 11 g/dL, LDH 500 U/L, INR 3. Esta entidad se produce por acumulación de: Hierro Cobre Esfingolípidos Alfa-1-antitripsina Se trata de un caso de falla hepática aguda muy probablemente secundaria a enfermedad de Wilson (historia familiar de muerte por falla hepática aguda, anemia hemolítica, fosfatasa alcalina baja, AST>ALT). La enfermedad de Wilson se produce por acumulación de cobre.

¿Cuál de los siguientes genes se encuentra involucrado en la patogénesis de esta enfermedad? HFE C282Y H63D ATP7B Las otras opciones son alelos alterados en hemocromatosis y el gen HFE es la principal causa de hemocromatosis.

¿Cuál de las siguientes manifestaciones sugiere otra patología? Anemia hemolítica Anillo de Kayser-Fleischer Diabetes y cardiomiopatía por depósito Elevaciones leves de las transaminasas con fosfatasa alcalina baja La diabetes y cardiomiopatía son manifestaciones típicas de hemocromatosis (acúmulo de hierro) no por enfermedad de Wilson.

¿Cuál es el mejor tratamiento para este paciente? Trientina D-penicilamina Trasplante hepático

Zinc La falla hepática aguda en pacientes con enfermedad de Wilson tiene prácticamente 100% de mortalidad en ausencia de trasplante hepático. El resto son tratamientos utilizados en pacientes diagnosticados con formas crónicas de la enfermedad.

¿Cuál es la explicación más probable de las manifestaciones neurológicas en su hermana? Depósito de cobre en los ganglios basales Edema cerebral Depósito de hierro en el núcleo lenticular Isquemia cerebral por vasculitis Los trastornos del movimiento en pacientes con enfermedad de Wilson son secundarios a depósito de cobre en el núcleo lenticular. Es más frecuente encontrar anillo de Kayser-Fleischer en pacientes con manifestaciones neurológicas.

CASO CLÍNICO 4 Hombre de 60 años de edad con historia de ingesta de alcohol en exceso, suspendido hace 10 años (tomaba aproximadamente 80 gramos de alcohol al día). Se encuentra asintomático aunque tiene la duda de si tiene cirrosis hepática por lo que acude para consulta de rutina. ¿Cuál hallazgo descartaría cirrosis hepática? Trombocitopenia Eritema palmar Trombocitosis Telangiectasias en la mitad superior del cuerpo La trombocitosis no es un hallazgo típico en un paciente con cirrosis hepática.

Debido a que la valoración clínica no fue concluyente, usted decidió realizar un ultrasonido hepático que mostró hígado macroscópicamente normal con los bordes discretamente lobulados y bazo de tamaño normal. Se realizó también elastografía hepática de transición que reportó fibrosis “F4”. De acuerdo a estos hallazgos, ¿Qué estudio auxiliar es irrelevante en el seguimiento del paciente? Esofagogastroduodenoscopia Ultrasonido hepático semestral Inmunización para VHA y VHB Alfafetoproteína semestral La alfafetoproteína no tiene ningún papel actualmente en el tamizaje de hepatocarcinoma por su baja sensibilidad.

Pasados 6 meses, se solicitó estudio de imagen, el cuál mostró una lesión en lóbulo hepático derecho de 2 cm de diámetro, con reforzamiento arterial y lavado en fases tardías. De acuerdo a los hallazgos, ¿cuál de los siguientes es el diagnóstico diferencial más apropiado?

Hepatocarcinoma fibrolamelar Hepatocarcinoma Colangiocarcinoma Hiperplasia nodular focal Se trata de una lesión que tiene comportamiento típico por imagen de hepatocarcinoma lo cuál es diagnóstico en pacientes con cirrosis sin necesidad de obtener biopsia de la lesión.

Además se realizó estudio endoscópico, que mostró várices esofágicas grandes con datos de mal pronóstico. Nunca ha tenido episodios de hemorragia. De acuerdo a esto, podríamos inferir que si a este paciente se le practicara un gradiente de presión venosa hepática, ¿en cuál de los siguientes rangos de presión se encontraría? 5-10 mmHg 10-12 mmHg >12 mmHg 15-20 mmHg La presión portal normal es 6-7 g/dL Hb >10 g/dL Hb normal Se prefieren estrategias de transfusión restrictivas. Al parecer cifras de hemoglobina >8 confieren mayor riesgo de resangrado por lo que no debe llevarse la hemoglobina a >8 g/dL.

Se realizó endoscopia, encontrando 3 paquetes de várices esofágicas grandes con puntos rojos. Una de ellas con un pezón blanquecino como estigma de sangrado reciente. Se realizó ligadura de las várices. Su evolución fue favorable y fue egresada. Cuatro meses después acude por incremento progresivo del perímetro abdominal y edema de miembros inferiores. Niega fiebre o dolor abdominal. Exploración física: con matidez cambiante y ascitis grado 2. Se realizó paracentesis encontrando Albúmina de 1 g/dL (albúmina sérica de 3.5 g/dL), neutrófilos de 350 por campo, proteínas totales de 1.2 g/dL. De acuerdo a estos hallazgos, ¿cuál es la conducta a seguir?

Realizar tomografía de perforación Vigilancia Ceftriaxona IV + Albúmina Furosemida IV + albúmina

abdomen

para

buscar

Se trata de una ascitis neutrocítica. Dicha entidad tiene 100% de mortalidad sin tratamiento por lo que es mandatorio iniciar antibióticos (son de elección las cefalosporinas de tercera generación). Se recomienda además dar albúmina a 1.5 gramos por kg el día 1 y a 1 gramo por kg el día 2.

Su evolución fue favorable y egresó a los 5 días con tratamiento a base de diuréticos. Dos semanas después, acudió nuevamente a urgencias por fatiga. Los laboratorios reportaron Cr 2 mg/dL (0.8 hace dos semanas). Se realizó sedimento urinario que no reportó alteraciones (blando), ultrasonido renal que fue reportado normal. En el examen general de orina no hay proteinuria significativa. Se calculó la Fracción excretada de sodio (FeNa=0.1). ¿Cuál es la conducta más apropiada en este momento? Suspensión de diuréticos, albúmina a 1 gr/kg por 48 horas y revaloración Iniciar albúmina y terlipresina IV Iniciar albúmina y norepinefrina IV Subir dosis de diuréticos para evitar sobrecarga hídrica

En un paciente con cirrosis y lesión renal aguda, antes de integrar el diagnóstico de síndrome hepatorrenal deben corregirse causas de depleción de volumen (suspensión de diuréticos) y realizar expansión de volumen con albúmina. Si después de 48 horas de albúmina no hay mejoría y se descartan causas secundarias (obstrucción urinaria, necrosis tubular aguda, glomerulopatías) puede integrarse el diagnóstico de síndrome hepatorrenal.

Durante la hospitalización, la paciente presentó somnolencia, asterixis y rueda dentada. ¿Cuál de las siguientes conductas no es apropiada en este momento? Descartar desequilibrio electrolítico Revisar cuantas evacuaciones ha reportado enfermería Iniciar lactulosa vía oral Realizar tomografía de cráneo No es necesario realizar TAC de cráneo ya que la paciente no tiene antecedentes o datos que orienten a la presencia de alguna entidad intracraneal.

CASO CLÍNICO 6 Mujer de 22 años de edad previamente sana. Tuvo hace 2 meses un episodio de colecistitis aguda que requirió colecistectomía resuelta sin complicaciones, sin embargo en el ultrasonido se documentó de forma incidental una lesión en lóbulo hepático derecho de 2 cm de diámetro. No ha tenido síntomas, sin embargo se encuentra preocupada y acude a consulta. ¿Cuál es la mejor conducta a seguir en este momento? Vigilancia con ultrasonido semestral Colangiorresonancia magnética Tomografía trifásica de hígado Cirugía con hepatectomía derecha El abordaje inicial de las lesiones hepáticas ya diagnosticadas mediante USG, consiste en estudios de imagen dinámicos en múltiples fases (arterial, portal, venosa y tardía).

La paciente acudió a seguimiento pero olvidó el reporte. Solo recuerda que la hoja decía que tenía la lesión hepática benigna más frecuente. ¿Cuál de los siguientes hallazgos se espera que el radiólogo haya observado? Reforzamiento arterial y lavado venoso tardío Reforzamiento periférico y llenado centrípeto tardío Reforzamiento venoso Lesión hepática rodeada de múltiples lesiones satélites e hígado nodular difuso La lesión benigna más frecuente de hígado es el hemangioma y el hallazgo típico es el reforzamiento periférico y llenado centrípeto tardío.

¿Cuál es la conducta más apropiada en este momento? Realizar biopsia Enviar a cirugía para resección Realizar resonancia con medio de contraste Vigilancia expectante El estudio de imagen razonablemente no da un diagnóstico y la biopsia de una lesión vascular confiere alto riesgo de sangrado por lo que por el momento sólo requiere vigilancia.

La paciente se quedó inquieta y acudió con otro médico quien realizó biopsia. ¿Cuál de los hallazgos histopatológicos es más probable en la entidad en cuestión? Cordones de hepatocitos maduros sin triadas portales Espacios vasculares rodeados de células endoteliales con estroma fino Células poco diferenciadas en estructuras glandulares Hepatocitos que forman nódulos de regeneración sin fibrosis circundante La presencia de cordones de hepatocitos maduros sin triadas portales es sugerente de adenoma, la presencia de células poco diferenciadas que forman estructuras glandulares es sugestiva de metástasis hepáticas de adenocarcinoma y la presencia de hepatocitos que forman nódulos de regeneración sin fibrosis es sugerente de hiperplasia nodular regenerativa.

La paciente toma anticonceptivos y desea embarazarse. ¿Cuál de las siguientes opciones es la más apropiada respecto al uso de hormonales en esta paciente? Existe contraindicación absoluta para su embarazo Puede continuar anticonceptivos hasta que se embarace y el embarazo es de riesgo estándar Debe resecarse la lesión antes del embarazo Se trata de un embarazo de alto riesgo y de preferencia esperar 2 a 3 años para planearlo El único tumor hepático que requiere cambio respecto a la conducta en el manejo de hormonales y/o embarazo es el adenoma, pero no es el caso.

Colelitiasis / Pancreatitis aguda y crónica / Tumores de páncreas CASO CLÍNICO 1 Mujer de 40 años de edad previamente sana. Inició hace un año con dolor en hipocondrio derecho postprandial intermitente (aproximadamente un episodio al mes), intenso, con irradiación a escápula derecha y acompañado de náusea y ocasionalmente vómito de contenido gástrico. La paciente no desea operarse.

¿Cuál es la probabilidad de desarrollar complicaciones y la probabilidad de recurrencia de episodios de dolor similares a los que ha tenido? 10-15% complicaciones / 5-10% recurrencia de dolor 20-30% complicaciones / 30-40% recurrencia de dolor 2-3% complicaciones / 30-40% recurrencia del dolor 5-6% complicaciones / 20-30% recurrencia del dolor Una vez que un paciente tuvo episodios de cólico biliar tiene un riesgo de 23% de desarrollar complicaciones y 30-40% de recurrencia de nuevos episodios. La tasa de progresión de colelitiasis asintomática a sintomática es de 2% por año por eso no está justificado el tratamiento quirúrgico como primera opción.

La paciente decidió no operarse y acudió 3 meses después por dolor en hipocondrio derecho, ictericia y fiebre. Aun no desea operarse y solo requiere algún tratamiento que le resuelva el episodio actual. Exploración física: taquicárdica. Los exámenes de laboratorio muestran BT 4.5 mg/dL, BD 3.5 mg/dL, BI 1 mg/dL, FA 200 U/L. Leucocitos de 18000 x µL. El ultrasonido de hígado y vías biliares muestra dilatación de VB hasta 8 mm. ¿Cuál es la mejor conducta terapéutica en este momento? Colangiorresonancia y si es positiva CPRE CPRE desde el inicio Ultrasonido endoscópico y si es positivo CPRE Colecistectomía con exploración de la vía biliar La paciente tiene dos predictores muy fuertes de coledocolitiasis (colangitis y BT >4 mg/dL) por lo que la probabilidad es alta. En estos casos el tratamiento de elección es CPRE. Actualmente se considera la CPRE como el estándar de tratamiento de litiasis de la VB antes de la colecistectomía por su menor invasividad.

Fue tratada exitosamente por su grupo médico y fue egresada, sin embargo, 6 meses después, acude nuevamente al servicio de urgencias por dolor en hipocondrio derecho de 8 horas de evolución, intenso, acompañado de fiebre en las últimas 2 horas. Se realizó ultrasonido de hígado y vías biliares que mostró vesícula biliar distendida con engrosamiento de su pared de 5 mm y signo de Murphy ultrasonográfico. ¿Cuál es la fisiopatogenia del cuadro clínico actual de la paciente? Obstrucción del colédoco por lito Obstrucción transitoria del cístico por lito Obstrucción persistente del cístico por lito Infección por anaerobios productores de gas La fisiopatología es obstrucción persistente del cístico por un lito y en algunos casos infección bacteriana secundaria. Cuando la vesícula se infecta por anaerobios productores de gas, se produce colecistitis enfisematosa lo cual es poco frecuente aunque grave.

Durante la cirugía se encuentra vesícula biliar friable con múltiples adherencias circundantes y una fístula colecistoduodenal. Se realiza cierre primario y colecistectomía sin complicaciones. ¿Cuál de las siguientes entidades se asocia a la presencia de una fístula colecistoduodenal? Síndrome de Mirizzi Colecistitis enfisematosa Íleo biliar Coledocolitiasis Para que sea posible una oclusión intestinal por litos, es necesario que exista una fístula colecistoduodenal ya que los litos obstructivos usualmente son grandes (2.5 cm) y no pueden pasar por el colédoco. Los sitios más frecuentes de obstrucción son el íleon terminal y la salida gástrica, este último llamado síndrome de Bouveret.

En el postoperatorio, la paciente presentó fiebre, dolor e ictericia. Los exámenes de laboratorio muestran BD 3.5 mg/dL, FA 300 U/L y el ultrasonido muestra dilatación de la vía biliar hasta 10 mm. La paciente no deseó ningún tratamiento adicional y 24 horas después, inició con deterioro del estado de alerta e inestabilidad hemodinámica. Los exámenes de laboratorio muestran BT 6 mg/dL, BD 5 mg/dL, leucocitos de 25000 x µL. ¿Cuál es la mejor conducta a seguir? Cirugía urgente CPRE urgente Drenaje biliar percutáneo Antibióticos y tratamiento conservador Se trata de una colangitis grave con Péntada de Reynolds que requiere drenaje urgente de la vía biliar. El método de elección es CPRE. El drenaje biliar percutáneo se reserva para los casos de CPRE fallida.

CASO CLÍNICO 2 Mujer de 40 años de edad, previamente sana y sin antecedentes de importancia. Acude al servicio de urgencias por dolor abdominal en epigastrio, intenso, transfictivo con irradiación en hemicinturón hacia la espalda. Los exámenes de laboratorio son normales con excepción de la lipasa, que se reporta en 1100 U/L. Esta indicación la aplicas si has descartado las causas más comunes: Hidratación vigorosa Realizar ultrasonido de hígado y vías biliares Realizar tomografía de abdomen Ayuno y analgesia En este caso se trata de un cuadro típico por lo que el papel de la tomografía en estos casos es limitado. Debe reservarse para pacientes que no mejoran

después de 48 a 72 horas y cuando se descartaron causas comunes mediante el USG y exámenes de laboratorio.

¿Cuál de las siguientes es la forma más apropiada para reanimar a la paciente? Solución glucosada al 5% 5-10 mL/Kg/hr Solución mixta 5-10 mL/Kg/hr Solución salina al 0.9% 5-10 mL/kg/hr Solución Ringer lactato 5-10 mL/kg/hr La solución de elección es Ringer lactato hasta lograr las metas de resolución. El requerimiento promedio de volumen en un paciente con estas características es de 2.5 a 4L. Hay que llevar el hematocrito a 44%.

La paciente ha evolucionado favorablemente después de 48 horas de estancia hospitalaria. Actualmente ya no tiene dolor. ¿Cuál es la conducta más apropiada en este momento? Iniciar nutrición enteral Iniciar vía oral Iniciar nutrición parenteral Iniciar vía oral previa medición de enzimas pancreáticas Se trata de un caso de pancreatitis aguda leve. Este grupo de pacientes puede ser alimentado por vía oral en cuanto se resuelva el dolor.

Este hallazgo te hablaría de complicación de la paciente: Edema pancreático en la tomografía Creatinina de 0.8 a las 48 horas Índice de Kirby de >400 a las 48 horas Signo de Grey-Turner El signo de Grey-Turner es un dato de pancreatitis necrótico-hemorrágica por lo que esperaríamos ver a un paciente con falla orgánica persistente y otros parámetros de gravedad. El resto puede verse en pacientes con pancreatitis leve como en este caso.

Se solicitó un estudio de imagen que mostró la presencia de múltiples litos en la vesícula biliar. No hay dilatación de la vía biliar. Las pruebas de función hepática son normales. ¿Cuál es la mejor conducta a seguir en esta paciente? CPRE con esfinterotomía y colecistectomía electiva CPRE y colecistectomía en esta hospitalización Colecistectomía durante esta hospitalización Ultrasonido endoscópico y si es positivo, CPRE con posterior colecistectomía durante hospitalización A pesar de que las PFH son normales y no hay dilatación de la vía biliar, la paciente tiene un predictor moderado de colédoco-litiasis (pancreatitis biliar). Lo ideal es descartar coledocolitiasis mediante un método no invasivo (colangiorresonancia o ultrasonido endoscópico) y CPRE sólo en caso de ser positivo. Por otro lado, en pacientes con pancreatitis leve, se recomienda colecistectomía antes del alta.

CASO CLÍNICO 3 Hombre de 60 años de edad diabético y con obesidad mórbida. Inició hace 2 horas con dolor abdominal epigástrico, transfictivo con irradiación hacia hipocondrio izquierdo por lo que acudió al servicio de urgencias. Exploración física: paciente taquicárdico y con fiebre (38.5°C). La saturación de oxígeno al aire ambiente es de 85% por lo que se pasa a área de hospitalización y se inicia oxígeno con puntas nasales con corrección de la hipoxemia. Los exámenes de laboratorio muestran BT 5 mg/dL, BD 4 mg/dL, BI 1 mg/dL, FA 200 U/L, ALT 200 U/L, AST 185 U/L. Amilasa 1000 U/L, triglicéridos 500 mg/dL. Calcio corregido de 11 mg/dL. ¿Cuál es la causa más probable de la pancreatitis aguda en este paciente? Hipertrigliceridemia Biliar Alcohol Hipercalcemia La hipertransaminasemia y el resto de alteraciones de las PFH son sugestivas de pancreatitis biliar. El nivel de triglicéridos descrito para pancreatitis es >1000 mg/dL. El paciente niega ingesta de alcohol y no hay algún otro dato orientador al respecto.

Dos horas después, inicia además con dolor en hipocondrio derecho y nuevo pico febril acompañado de escalofríos. Se documenta hipotensión con TA de 80/50mmHg que responde a líquidos. Además de continuar con la reanimación, ¿cuál es la conducta más apropiada en este momento? Antibióticos y vigilancia CPRE y antibióticos Iniciar aminas Realizar TAC y normar conducta a seguir de acuerdo a los hallazgos Se trata de una pancreatitis aguda con colangitis, que es una de las pocas indicaciones de CPRE temprana en pacientes con pancreatitis aguda (la otra indicación es colestasis persistente).

Se lleva a cabo el manejo que usted indicó, sin embargo el paciente persiste con datos de respuesta inflamatoria y a las 48 horas presenta hipoxemia refractaria que requiere intubación endotraqueal. ¿Cuál de los siguientes parámetros clasificaría a este paciente como pancreatitis aguda grave? Hipoxemia persistente Hipoxemia refractaria Obesidad mórbida y episodio de hipotensión transitorio Hipotensión transitoria

La definición actual de pancreatitis aguda grave es la presencia de falla orgánica persistente por >48 horas de acuerdo a los criterios de Marshall. La falla orgánica transitoria y/o la presencia de complicaciones locales o exacerbaciones de comorbilidades previas definen a la pancreatitis moderada.

¿Cuál es la mejor forma de iniciar el tratamiento nutricional en este paciente? Mantener sin aporte calórico por los primeros 7 días y después revalorar Nutrición parenteral total Nutrición enteral Vía oral La nutrición enteral temprana se asocia con mejores desenlaces en pancreatitis aguda. La nutrición parenteral se reserva para los casos de pobre tolerancia o bien cuando hay alguna condición que impida el uso del tracto digestivo.

El paciente ha evolucionado favorablemente, sin embargo, hace 1 día, se realizó tomografía de abdomen que mostró necrosis no encapsulada de 30% del parénquima pancreático hacia el cuerpo y cola sin gas. El paciente no ha tenido datos de respuesta inflamatoria sistémica ¿Cuál es la mejor conducta en este momento respecto a la necrosis? Drenaje percutáneo de la necrosis Vigilancia expectante Punción de la necrosis e iniciar antibióticos si es positiva Iniciar antibióticos profilácticos En pacientes con pancreatitis aguda y colecciones se prefiere el tratamiento más conservador posible, en especial cuando son colecciones tempranas.

CASO CLÍNICO 4 Hombre de 55 años, ex alcohólico actualmente. Inició hace 2 meses con diarrea con esteatorrea y lientería en abundante cantidad, acompañada de pérdida de 5 kg de peso durante este período. Ha tenido episodios de dolor epigástrico transfictivo de moderada intensidad que lo incapacitan de sus actividades cotidianas. Solicitaste estudio de imagen que mostró páncreas atrófico con múltiples calcificaciones y litos intraductales con dilatación del conducto pancreático hasta 7 mm. ¿Cuál es la causa más frecuente de la enfermedad de este paciente? Biliar Hipertrigliceridemia Tabaquismo Alcohol La causa más frecuente de pancreatitis crónica es alcohol.

La patología citada no es un factor de riesgo para: Adenocarcinoma de páncreas Obstrucción de la vía biliar Obstrucción del duodeno Cistadenomas serosos de páncreas La pancreatitis crónica no es un factor de riesgo para desarrollo de cistadenomas serosos de páncreas. Como complicaciones puede ocasionar obstrucción del biliar y duodenal y en un porcentaje pequeño, adenocarcinoma de páncreas.

¿Cuál es el tratamiento inicial recomendado para la diarrea malabsortiva en este paciente? Descompresión del conducto pancreático Enzimas pancreáticas sin cubierta entérica Enzimas pancreáticas con cubierta entérica Antibióticos El tratamiento inicial de elección para la insuficiencia exocrina es lipasa pancreática, por lo menos 30000 U diarias.

El paciente ha mantenido seguimiento en la consulta externa, sin embargo en los últimos meses ha tenido aumento progresivo del dolor. ¿Cuál efecto adverso no se relaciona con el tratamiento médico de elección en el control a largo plazo del dolor moderado a grave en pancreatitis crónica? Constipación Somnolencia Úlcera péptica Dependencia Los efectos adversos son dismotilidad intestinal (constipación, retraso en el vaciamiento gástrico), dependiendo del opiáceo los efectos cognitivos son variables pero pueden causar somnolencia y finalmente, el efecto más indeseable que limita el uso de opiáceos en pancreatitis crónica es el desarrollo de tolerancia/dependencia y adicción a opiáceos en algunos pacientes.

Se optimizó el tratamiento médico del paciente, sin embargo continúa con dolor. Se realizó nueva tomografía, encontrando los litos intraductales previamente observados con dilatación del conducto pancreático a 8 mm sin otras alteraciones. Con esta información, de los siguientes, ¿Cuál es el mejor tratamiento a ofrecer en este momento? Pancreatoduodenectomía (Whipple) Pancreatectomía-total Pancreatectomía distal con esplenectomía Pancreatoyeyuno anastomosis (Puestow) En general en pancreatitis crónica, siempre se prefiere conservar la mayor parte de parénquima pancreático. Los pacientes con dilatación del conducto pancreático son los mejores candidatos a procedimientos derivativos ya que su anatomía permite hacer la pancreatoyeyuno anastomosis laterolateral con descompresión del parénquima pancreático. Se ha comparado tratamiento quirúrgico vs endoscópico y a corto plazo tienen eficacia similar

para control del dolor pero a largo plazo, la efectividad del tratamiento quirúrgico es mayor.

CASO CLÍNICO 5 Mujer de 60 años de edad. Hace 1 mes acudió a una revisión con su gastroenterólogo, quien solicitó una tomografía abdominal, encontrando una lesión quística en la cabeza del páncreas de 2 cm de diámetro con patrón en panal de abejas. La describen como una lesión lobulada con múltiples microquistes en su interior con una cicatriz central. No hay dilatación del conducto pancreático ni de la vía biliar. ¿Cuál es el siguiente paso en el abordaje diagnóstico/terapéutico en esta paciente? Ultrasonido endoscópico con biopsia Pancreatografía endoscópica Resección de la lesión Vigilancia expectante Con el estudio de imagen realizado, queda claro que se trata de un cistadenoma seroso de páncreas el cual tiene un extraordinariamente bajo potencial maligno. Se encuentra en una localización típica de este tipo de tumores (cabeza), su morfología por imagen es típica (microquistes y cicatriz central), no hay síntomas y no hay datos de obstrucción por lo que la paciente puede ser vigilada sin necesidad de otro estudio diagnóstico.

¿Cuál es el potencial de transformación maligna de la neoplasia quística en esta paciente? 0-1% 5% 10% 30% El potencial maligno es prácticamente nulo y esta es una de las razones por las que se recomienda tratamiento expectante.

La hermana de la paciente de 52 años, se quedó preocupada y ella también se realizó una tomografía que mostró una lesión quística en la unión del cuerpo del páncreas con la cola. Dicha lesión tiene un diámetro de 3 cm, es de aspecto macroquístico y tiene calcificación periférica. Al interrogatorio refiere que se encuentra asintomática. ¿Cuál de las siguientes opciones es característica exclusiva en esta lesión? Tiene potencial maligno y debe resecarse Se comunica con el conducto pancreático Solo las mujeres son afectadas por esto tumores Su principal característica histológica es la presencia de estroma ovárico Se trata de un tumor mucinoso que tiene aspecto típico por imagen (macroquístico con calcificación periférica en cáscara de huevo). Este grupo de tumores son casi exclusivos de mujeres.

La paciente es médico y no desea tratarse e insiste en la realización de un ultrasonido endoscópico con aspiración. ¿Cuál de los siguientes hallazgos es característico en este tipo de lesiones? Elevación del antígeno carcinoembrionario en el líquido Elevación del Ca-19-9 en el líquido Elevación de la amilasa en el líquido Mucina ausente El marcador de neoplasias mucinosas en el líquido de aspiración es el antígeno carcinoembrionario. Niveles altos sugiere una neoplasia mucinosa.

Se realiza el tratamiento indicado y permanece asintomática por lo que no acudió ya a más consultas de seguimiento. Regresa con usted 10 años después debido a que en una tomografía realizada le detectaron dilatación del conducto pancreático de 5 mm. Le solicita una pancreatografía por resonancia magnética que mostró una lesión de 1 cm de diámetro en el conducto pancreático principal, en la cabeza del páncreas. ¿Cuál de las siguientes no es una característica típica de este tipo de lesiones? Producen moco Tienen potencial maligno y deben resecarse Pueden producir episodios de pancreatitis aguda Tienen niveles bajos de antígeno carcinoembrionario en el líquido Se trata de una neoplasia papilar intraductal mucinosa en inglés IPMN. Al ser neoplasias mucinosas, se espera un contenido elevado de ACE en el líquido. Debido a obstrucción del sistema ductal, tienen el potencial de presentarse con episodios de pancreatitis aguda. Por su potencial maligno deben resecarse.

CASO CLÍNICO 6 Hombre de 70 años con historia de consumo excesivo de alcohol y tabaquismo con IT 30. (Ambos actualmente suspendidos). Inició hace 3 meses con dolor epigástrico intermitente, tolerable con irradiación a hipocondrio izquierdo. Al interrogatorio dirigido refiere pérdida de 12 kg de peso desde el inicio de su padecimiento. Se indicó realización de tomografía de abdomen que mostró una lesión mal definida en la cabeza del páncreas de 2 cm con dilatación del conducto pancreático de 5 mm, dilatación del colédoco y páncreas atrófico en cuerpo y cola.

¿Cuál de los siguientes se desconoce como un factor de riesgo para el desarrollo de este tipo de neoplasias? Pancreatitis crónica Tabaquismo Neoplasia intraductal papilar mucinosa Edad 20-30 años Estos tumores típicamente se observan en pacientes de edad avanzada.

¿Cuál de las siguientes no es una característica de este tipo de neoplasias? La mayoría son resecables al diagnóstico Tienen una alta tasa de mortalidad El tipo más frecuente es el adenocarcinoma ductal Los tumores de la cabeza causan obstrucción de la vía biliar La mayoría de estos tumores se presentan en etapas avanzadas al diagnóstico y son irresecables, sólo 10 a 20% son resecables. El adenocarcinoma ductal (el tipo más frecuente) es uno de los cánceres más letales. Los tumores de la cabeza tienden a presentarse antes que los del cuerpo y cola debido a que causan ictericia y dilatación de la vía biliar.

Usted decidió solicitar una tomografía trifásica de páncreas, la cual reportó una tumoración en la cabeza del páncreas de 2.2 cm con signo del doble conducto. La tumoración pierde interfase con la arteria mesentérica superior en la mayor parte de su circunferencia. Se realizó ultrasonido endoscópico con biopsia que mostró glándulas atípicas con invasión del estroma y abundante reacción desmoplásica. De acuerdo a estas características, ¿cuál es el mejor tratamiento en este momento? Cirugía de Whipple Colocación de prótesis biliar y posteriormente cirugía de Whipple Quimioterapia Neurolisis del plexo celiaco Se trata de una lesión irresecable en la cabeza del páncreas que tiene confirmación histológica de adenocarcinoma por lo tanto no puede ofrecerse ninguna opción quirúrgica. En estos casos el tratamiento a seguir es quimioterapia/paliación de síntomas. El tratamiento del dolor inicialmente es médico.

¿Qué fármacos se encuentra fuera de los quimioterapéuticos utilizados para este tipo de neoplasias? Gemcitabina Oxaliplatino Fluoracilo L-asparaginasa Las principales quimioterapias utilizadas son la gemcitabina y el esquema FOLFIRINOX (Ácido Folínico, Fluoracilo, Irinotecan, Oxaliplatino). La Lasparaginasa no tiene ningún papel.

En el caso de tumores resecables que se encuentran en la cabeza del páncreas, ¿Cuál es el procedimiento de elección? Pancreatoduodenectomía Resección de la cabeza del páncreas Pancreatectomía y esplenectomía Pancreatectomía distal El estándar de tratamiento quirúrgico estándar para los tumores de la cabeza de páncreas, es la pancreatoduodenectomía (Cirugía de Whipple). Dicha cirugía puede o no preservar el píloro y requiere reconstrucción de la vía biliar (hepato-yeyuno anastomosis), anastomosis del páncreas residual con el intestino (pancreatoyeyuno anastomosis) y Reconstrucción del tubo digestivo (gastroyeyuno o duodeno-yeyuno anastomosis).

FINAL CLINICAL CASE 1 A 60-year-old man is referred with a 10-year history of gastro esophageal reflux for which he has been taking pantoprazol 40 mg orally 30 minutes before breakfast every day. He has been asymptomatic since he began the PPI therapy. He smokes 4 cigarettes per day since he was 20. What is the next step in the management of this patient? Continue PPI therapy with the same dose Perform an upper endoscopy Perform and upper endoscopy only if the patient develops alarm symptoms Performa an ambulatory 24-hour pH monitoring Consider surgical treatment for gastro esophageal reflux It’s a patient with multiple risk factors for Barrett esophagus. In general there’s no role for Barrett’s screening except for men older than 50 years with more than 5 years of GERD which are obese or smoke. The presence of this factors are all indications for screening.

The following are risk factors for developing Barrett’s esophagus, except: Men Age >50 years Tobacco Alcohol BMI As previously stated all of the listed factors in the options are risk factors except alcohol.

Which of the following make the diagnosis of Barrett`s esophagus: Intestinal metaplasia Atrophy Ulcer

Dysplasia None of the above With endoscopic view, one can have a general picture if the patient has or no Barrett esophagus but definitive diagnosis is made by histology.

In addition to the main finding that you selected in the previous question, which of the following histologic findings is mandatory for the diagnosis of Barrett’s esophagus? Presence of globet cells Heterotopic gastric mucosa Ectopic pancreatic tissue Lymphoid infiltrates PMN infiltrates The presence of globet cells is mandatory for the pathologist for the definitive diagnosis.

Multiples biopsies revealed Barrett’s esophagus without dysplasia. Then you decide: Endoscopy surveillance PPI therapy for symptoms Sucralfate A and B are correct B and C are correct Patients with Barrett, need long term endoscopic surveillance. The periodicity of this depends on the presence or absence of dysplasia. Patients with dysplasia need more aggressive surveillance. Also this patients need lifelong PPI therapy.

CASO CLÍNICO 2 Mujer de 70 años de edad con hipertensión arterial sistémica en tratamiento con amlodipino. Desde hace 6 meses presenta dolor abdominal epigástrico, plenitud temprana, edema de miembros inferiores. Sus laboratorios con Hb 13 g/dl, Cr 1 mg/dl, albúmina 2 g/dl, EGO sin proteinuria. Se le realizó una panendoscopia en la que se observó engrosamiento de pliegues gástricos. ¿Cuál es el diagnóstico más probable? Adenocarcinoma gástrico Linfoma gástrico Síndrome de Zollinger-Ellison Gastroenteritis eosinofílica Enfermedad de Ménétrier El engrosamiento de pliegues gástricos aunado a la hipoalbuminemia de origen no renal sugiere una enteropatía perdedora de proteínas.

Con respecto a la hipoalbuminemia de esta paciente, es cierto que: Es secundaria a una enteropatía perdedora de proteínas Es por pérdidas renales y amerita recolección de orina de 24 horas

No tiene relación con la sospecha clínica B y C son correctas Tiene ego sin proteinuria y los hallazgos endoscópicos correlacionan con una probable enfermedad de Ménétrier.

¿Cuál de los siguientes es un diagnóstico diferencial? Cáncer gástrico Síndrome de Zollinger-Ellison Sarcoidosis Gastropatía hipertrófica asociada a H. pylori Todas las anteriores Todas las anteriores dan engrosamiento de pliegues gástricos similar a enfermedad de Ménétrier.

El diagnóstico definitivo de esta patología es: Hallazgos endoscópicos Biopsia de pared gástrica Por la hipoalbuminemia Diagnóstico de exclusión Ninguna de las anteriores La histología es indispensable para realizar este diagnóstico.

Tratamiento de esta patología es: Gastrectomía Vigilancia endoscópica IBP No amerita tratamiento A y B son correctas Tratamiento con gastrectomía por riesgo asociado de cáncer gástrico. Sin embargo se puede optar por vigilancia endoscópica regular.

CASO CLÍNICO 3 Paciente masculino con antecedente de úlcera péptica asociada a H. pylori documentada hace 2 meses, recibió tratamiento de erradicación. Se le tomaron biopsias gástricas que reportaron metaplasia intestinal incompleta en cuerpo y antro sin displasia. Actualmente se encuentra asintomático. ¿Cuál de las siguientes es una indicación de erradicación de Helicobacter pylori? Dispepsia Páncreas ectópico en estómago Linfoma gástrico y úlcera péptica Todas las anteriores A y C son correctas La indicaciones de erradicación más frecuentes son cáncer gástrico temprano, linfoma gastrointestinal, úlcera gástrica, úlcera duodenal y dispepsia.

Usted quiere saber si se logró la erradicación de H. pylori. ¿Qué estudio o estudios se pueden utilizar para corroborar esto? Serología para Helicobacter Prueba de aliento Panendoscopia con toma de biopsias A y B son correctas B y C son correctas Tanto la prueba de aliento como cultivo son de utilidad para corroborar erradicación de H. pylori. Se empieza por el menos invasivo. Sin embargo la pregunta está dirigida a todos los métodos para corroborar erradicación.

¿Cuál sería el riesgo de metaplasia intestinal incompleta a largo plazo? No existe ningún riesgo Úlcera péptica Perforación gástrica Cáncer gástrico B y C son correctas Metaplasia intestinal incompleta con sulfocianina + es el que mayor riesgo de cáncer gástrico.

¿Qué tipo de metaplasia es la que tiene mayor riesgo de cáncer gástrico? Metaplasia intestinal completa con sialomucina + Metaplasia intestinal incompleta con sialomucina Las dos presentan el mismo riesgo La metaplasia con sialomucinas + Metaplasia intestinal incompleta con sulfocianina + Metaplasia intestinal incompleta con sulfocianina + es el que mayor riesgo de cáncer gástrico.

En el seguimiento de este paciente será necesario: Realizar panendoscopia de vigilancia No amerita seguimiento si se corrobora erradicación de H. pylori El pepsinógeno sirve como marcador tumoral A y C son correctas Ninguna Tiene metaplasia intestinal incompleta y extensa (afecta dos áreas del estómago) por lo que amerita vigilancia por riesgo incrementado de adenocarcinoma.

CASO CLÍNICO 4 Masculino de 20 años de edad con DM1 sin complicaciones crónicas. Acude a consulta externa para abordaje de diarrea crónica compatible con malabsorción intestinal en base a cuantificación de grasas en 24 hr. La prueba de D-xylosa por debajo de los rangos adecuados.

Todas las siguientes puede ser causas de diarrea crónica en el paciente, excepto: Fibrosis quística Enfermedad Whipple Enfermedad de Crohn Enfermedad celiaca Sobrepoblación bacteriana Se trata de una diarrea malabsortiva originada en la pared intestinal por lo que los diagnósticos diferenciales solo deben incluir condiciones originadas en este sitio. La sobrepoblación bacteriana es una causa intraluminal sin embargo ocasiona disminución de la cifra de D-xilosa.

Su sospecha diagnóstica es: Enfermedad celiaca Sobrepoblación bacteriana Insuficiencia pancreática exocrina Diarrea por sales biliares Ninguna de las anteriores Usualmente la enfermedad celíaca se asocia a otras condiciones autoinmunes, entre ellas DM1. Los pacientes con DM de larga evolución pueden tener sobrepoblación bacteriana por dismotilidad intestinal, en este paciente es poco probable ya que no tiene complicaciones crónicas.

Ante la sospecha de enfermedad celiaca usted solicita inmunoglobulinas que fueron normales, el estudio de laboratorio con mayor rendimiento diagnóstico es: Anticuerpo antigliadina IgG Anticuerpo antigliadina IgA Anticuerpo antiendosmisio IgG Anticuerpo antitransglutaminasa por IgA Anticuerpo antitransglutaminasa IgG Los anticuerpos antitransglutaminasa IgA son de elección para el tamizaje de celíaca. Un 5% tienen deficiencia de IgA y en ellos la prueba de elección son los antigliadina IgG.

El resultado fue positivo. Esta enfermedad se asocia al siguiente HLA: DQ2 DR4 DQ6 DR8 Ninguno de los anteriores Los anticuerpos DQ2 y DQ8 se asocian a celíaca, y su ausencia tiene un valor predictivo negativo muy alto para descartar esta enfermedad.

En la biopsia de duodeno usted espera encontrar lo siguiente, excepto. Infiltrado intraepitelial por linfocitos Atrofia de vellosidades Aplanamiento de vellosidades Infiltrado por polimorfonucleares Hiperplasia de criptas El infiltrado típico en enfermedad celíaca es linfocítico.

CASO CLÍNICO 5 Hombre de 55 años de edad con antecedente de ingesta excesiva de alcohol. Refiere que toma aproximadamente 375 mL de ron al día desde hace aproximadamente 20 años. Acude al servicio de urgencias por ictericia. De acuerdo a estos datos. ¿Qué cantidad de alcohol en gramos toma diariamente? 60 gramos 40 gramos 70 gramos 90 gramos 110 gramos La cantidad ingerida de alcohol en gramos se obtiene al multiplicar los mL de la bebida consumida (en este caso ron) por su porcentaje de alcohol (usualmente 0.38) por 0.78 (un mL de alcohol pesa 1 gramo).

¿Cuál de las siguientes opciones representa mejor la cantidad diaria máxima recomendada de consumo de alcohol? 20 gramos de alcohol en hombres / 30 gramos de alcohol en mujeres 30 gramos de alcohol en ambos sexos 60 gramos de alcohol en hombres / 30 gramos de alcohol en mujeres 30 gramos de alcohol en hombres / 20 gramos de alcohol en mujeres 10 gramos de alcohol en hombres / 20 gramos de alcohol en mujeres La opción resaltada representa la cantidad diaria máxima recomendada de ingesta de alcohol. El hombre tiene una tolerancia mayor al daño hepático por alcohol que la mujer. A partir de 60 gramos de alcohol diario en hombres y aproximadamente 30-40g en mujeres, la presencia de esteatohepatitis alcohólica se eleva notablemente.

A la EF se encuentra ictérico con hipertrofia parotídea. Se solicitaron exámenes de laboratorio que mostraron BT de 22 mg/dL, AST 200 U/L, ALT 100 U/L, GGT 500 U/L, TP 22/12, creatinina 0.9 mg/Dl, Hb de 10 g/dL con VGM 69 fL, plaquetas 140 x 103. ¿Cuál de los hallazgos encontrados en el paciente NO es explicable por su ingesta excesiva de alcohol? Patrón de ALT/AST 1:2 Elevación de la GGT a 500 Hipertrofia parotídea Hiperbilirrubinemia de 22 VGM 69

En la presencia de ingesta excesiva de alcohol, se espera la presencia de macrocitosis. Cuando hay microcitosis como en este caso es mandatorio descartar causas de deficiencia de hierro (principalmente sangrado).

¿Cuál de los siguientes hallazgos histológicos no es probable encontrar en caso de que a este paciente se le realizara una biopsia hepática? Megamitocondrias Cuerpos de Mallory Infiltrado con polimorfonucleares Esteatosis macrovesicular Hepatitis de interfase Todas las opciones enlistadas son hallazgos característicos de las esteatohepatitis. Hay que recordar que a menos que el infiltrado neutrofílico sea muy intenso, al patólogo le es imposible distinguir entre esteatohepatitis alcohólica y no alcohólica. La hepatitis interfase es un hallazgo típico de hepatitis autoinmune.

Se calculó un puntaje de Maddrey de 68. De acuerdo a este valor, ¿cuál es la mejor conducta a seguir con este paciente? Pentoxifilina Prednisona Pentoxifilina + Prednisona Infliximab Plasmaféresis El tratamiento de elección en la hepatitis alcohólica grave (Maddrey >32) son los esteroides. Existen contraindicaciones para el uso de esteroides como hemorragia de tubo digestivo activa, infecciones graves, falla renal y en estos casos una alternativa es la pentoxifilina. La combinación de esteroides con pentoxifilina no tiene ningún papel.

CASO CLÍNICO 6 Mujer de 23 años sin antecedentes de importancia. Acude a consulta debido a dolor abdominal episódico inespecífico de 2 años de evolución. El dolor es casi siempre tipo cólico en mesogastrio, acompañado de distensión abdominal sin relación con las evacuaciones. En dos ocasiones ha tenido evacuaciones con hematoquezia en cantidad moderada. En los exámenes de laboratorio destaca Hb 11 g/dL, VGM 85, VSG 50 mm/h. Se realizó una tomografía de abdomen que mostró engrosamiento concéntrico del íleon terminal y la válvula ileocecal, coprocultivos y coproparasitoscópicos que fueron negativos. ¿Cuál es el diagnóstico más probable en este paciente? Colitis ulcerativa Ileítis por Y. enterocolítica Enfermedad celíaca Amebiasis intestinal Enfermedad de Crohn

Se trata de una paciente con dolor abdominal crónico, historia de sangrado de tubo digestivo y elevación de marcadores inflamatorios por lo que el primer diagnóstico diferencial es enfermedad inflamatoria intestinal. La enfermedad de Crohn es más probable en este caso debido a la afección segmentaria del íleon terminal.

De acuerdo a su diagnóstico presuncional, ¿cuál es el siguiente paso en el abordaje diagnóstico? Colonoscopia con ileoscopia Enteroscopia retrógrada Videocápsula endoscópica Coprocultivo para gérmenes poco usuales Entero Resonancia magnética Se trata de un cuadro con alta sospecha de enfermedad de Crohn. Recordando que la forma más frecuente de enfermedad de Crohn es la ileocolónica, el siguiente paso es colonoscopía con ileoscopia con toma de biopsias. Si bien el resto de estudios tiene cierto papel en algún punto del abordaje de la enfermedad de Crohn, no todos los pacientes los requieren.

¿Cuál de las siguientes no es una característica de esta enfermedad? Úlceras lineales profundas con áreas de mucosa sana intercaladas con zonas de mucosa enferma Afección universal del recto Elevación de títulos de ASCAS en algunos pacientes. Afección perianal en algunos pacientes Fístulas enterales en algunos pacientes La afección universal del recto es característica de CUCI no de Crohn. El resto de opciones enlistadas, corresponden a enfermedad de Crohn.

¿Cuál de los siguientes fármacos no es útil en el tratamiento a largo plazo de esta enfermedad? Azatioprina 6-mercaptopurina Prednisona Infliximab Metotrexate Los esteroides son útiles en la inducción de la remisión en enfermedad inflamatoria intestinal, no para el tratamiento de mantenimiento.

¿Cuál de las siguientes no es una manifestación extraintestinal típica de esta enfermedad? Eritema nodoso Pioderma gangrenoso Uveítis Artropatía axial Penfigoide ampolloso El penfigoide ampolloso no es una entidad asociada a la enfermedad inflamatoria intestinal.

CASO CLÍNICO 7 Mujer de 42 años de edad con diagnóstico reciente de hipercolesterolemia. Su último perfil de lípidos reportó

un colesterol total de 330 mg/dL. Inició hace 6 meses con fatiga y prurito el cuál en ocasiones es incapacitante. Su esposo refiere que ha notado cambio en la coloración de su piel. De acuerdo a los datos comentados, ¿Cuál de las siguientes manifestaciones clínicas no sería concordante con el cuadro de la paciente? Xantelasmas Xantomas Tiroiditis Boca seca Colangitis de repetición Se trata de un cuadro típico de colangitis biliar primaria. Dicha entidad se asocia con dislipidemia con hipercolesterolemia con xantomas y xantelasmas resultantes. La hepatopatía autoinmune asociada a colangitis de repetición es la colangitis esclerosante primaria.

Se realizaron exámenes de laboratorio que mostraron BT 1 mg/dL, ALT 35 U/L, AST 33 U/L, FA 525 U/L, albúmina 4 g/dL. El USG hepático fue reportado normal. De acuerdo a su primera sospecha diagnóstica ¿Cuál de los siguientes exámenes complementarios tiene mayor probabilidad de aportar información diagnóstica en esta paciente? Anticuerpos anti-nucleares Anticuerpos anti-mitocondriales Anticuerpos anti-músculo liso Anticuerpos anti-LKM 1 Anticuerpos anti-SLA El 95% de los pacientes con colangitis biliar primaria tienen anticuerpos antimitocondriales positivos.

El estudio previamente solicitado fue reportado positivo. ¿Cuál es el siguiente paso en el abordaje diagnóstico? Biopsia hepática Solicitar nivel de globulinas Con la información actual ya tenemos diagnóstico Anticuerpos anti-fosfolípidos Colangiorresonancia magnética Para integrar el diagnóstico de colangitis biliar primaria basta con cumplir 2 de 3 de los siguientes criterios: Elevación de fosfatasa alcalina o GGT, Anticuerpos antimitocondriales positivos, o hallazgos histológicos típicos (lesión ductal florida). Esta paciente ya tiene 2 de los criterios y no es necesario realizar más pruebas.

¿Cuál es el tratamiento de elección en este caso? No existe tratamiento específico Atorvastatina Prednisona Ácido Ursodesoxicólico

Esfinterotomía endoscópica El ácido ursodesoxicólico es el tratamiento de elección en los pacientes con colangitis biliar primaria y los pacientes que responden bioquímicamente tienen mejor pronóstico que aquellos sin respuesta.

¿Cuál de los siguientes tratamientos NO es útil en el manejo del prurito? Colestiramina Plasmaféresis Sertralina Rifampicina Prednisona Los esteroides no son útiles en el tratamiento del prurito por CBP. La primera elección para el tratamiento del prurito en CBP es la colestiramina. El resto de opciones puede utilizarse en algún punto del tratamiento. La plasmaféresis puede ser útil para el prurito refractario a tratamiento médico y en casos extremos el prurito per se es una indicación de trasplante de hígado.

CLINICAL CASE 8 A 60 year old woman with a history of longstanding HCV compensated cirrhosis, and a recent ankle sprain, presents to the emergency department with a 2 week history of abdominal swelling and lower limb edema. Physical examination reveals normal vital signs. Abdominal examination reveals caput medusae, diffuse abdominal distension with shifting dullness. There’s not pain upon palpation. She has never been told to have ascites. After the consult finishes the patient remembered that she has been taking NSAIDS for the ankle sprain for the last two weeks. Which of the following is the most useful clinical clue for the diagnosis of ascites? Fluid wave test Shifting dullness Flank percussion Protruding umbilical hernia Iceberg sign Which of the following is the best next step in the workup of this patient? Abdominal ultrasound Diagnostic paracentesis Abdominal CT Observe her response to diuretics TIPS The first step in the work up of any patient with ascites that has been recently diagnosed is paracentesis. The first tests to perform on the ascites samples are PMN count, albumin and total proteins. Once you have categorized the ascites as portal hypertensive vs non-portal hypertensive, the further workup must be individualized.

Which of the following is NOT a factor that contributes to new onset ascites in a patient with previously compensated cirrhosis? Acute kidney injury Hepatocellular carcinoma Portal vein thrombosis Progression of the primary liver disease Encephalopathy Encephalopathy, as ascites is another condition that defines decompensation in cirrhosis. Encephalopathy can coexist with ascites but by itself is not a factor that contributes to new onset ascites, rather is a marker of decompensation.

The ordered laboratory tests revealed Cr 1.6 mg/dL (two weeks ago 0.7 mg/L), FeNa 0.1. Ascites fluid samples revealed a PMN count of 125xhpf and the serum ascites albumin gradient was reported in 2 mg/dL. Which of the following options is not appropriate in the workup/treatment at this point? Albumin 1 g/kg Hold NSAID use Renal USG Urinary sediment Furosemide The first step in the management of acute kidney injury in cirrhosis is withholding any nephrotoxic drug. Diuretics cause azotemia primarily by volume depletion. Once nephrotoxic drugs are withheld, the next step is expansion of the intravascular volume with albumin (especially in those with stage 2 or 3 acute kidney injury). Parallel, it’s important to rule out other causes of acute kidney injury as obstructive renal failure (renal USG), acute tubular necrosis (sediment), Glomerulopathies (Sediment, erythrocyte and protein counts in urine).

After 48 hours of optimal medical management, the patient continue to deteriorate. Laboratories show Cr 3.5. You have ruled out structural causes of kidney injury and currently there’s no identified infection. Which of the following is not an appropriate option in the management of this patient? Albumin + terlipressin Albumin + norepinephrine Speed-up workup for liver transplant Paracentesis as needed for tense ascites Dialysis This patient has Type 1 hepatorrenal syndrome which is a form of acute kidney injury in cirrhotic patients characterized by peripheral vasodilation and intense renal vasoconstriction. The treatment of choice is a combination of a volume expander (albumin) plus vasopressors (Terlipressin or norepinephrine) to increase renal perfusion pressures. Dialysis has no role as an initial treatment for hepato-renal syndrome. Hepatorrenal syndrome has a very poor prognosis and medical treatments are only a bridge to transplant which is the most effective an also the only mean to achieve long term survival.

CLINICAL CASE 9 A 40 year old obese woman without relevant medical history, presents to emergency department with 2 hours of severe upper abdominal pain with radiation to the back. She reports that this is the worst pain ever and compares it to a knife that penetrates her abdomen towards her back. Since the beginning of her symptoms she has had about 5-6 episodes of copious bilious emesis. Her vital signs are HR 110, RR 22 and BP 110/70mmHg. Physical examination reveals tenderness to palpation with guarding. Which of the following is NOT an appropriate initial option in this patient diagnosis/management? Crystalloid infusion / opioid analgesia Nasogastric tube Abdominal USG Serum lipase Begin antibiotics There’s no role for prophylactic antibiotics in the absence of established infection in acute pancreatitis.

After 2 hours, with your initial management the patient appears more comfortable, her vitals are stable. Laboratory tests reveal creatinine 1 mg/dL, BUN 18 mg/dL, lipase 1200 U/L, Hematocrit 44%, Hemoglobin 13.5 g/dL, triglycerides 180 mg/dL, Bilirubin 5 mg/dL, Conjugated B 3.5 mg/dL, Unconjugated B 1.5 mg/dL, AST 300 UL, ALT 320 U/L, alkaline phosphatase 280 U/L. Which of the following is the most likely cause of the clinical picture in this patient? Persistent obstruction of the cystic duct by a stone Choledocolitiasis Pancreatic head mass Pancreatic ductal calcifications Pancreas divisum This patient clearly has pancreatitis (pain + 3x lipase elevation). The laboratory test abnormalities (cholestasis, moderate hypertransaminasemia) are suggestive of biliary tract obstruction.

After 6 hours the patient deteriorates. She begins with dyspnea and tachypnea. The pulse oximetry shows a SaO2 of 80% (room air). Because of accessory respiratory muscle use you decide to intubate the patient and transfer her to the ICU. Which of the following options is NOT true about current patient condition?

According to Atlanta classification she has acute severe pancreatitis There’s not enough information at this point to classify the severity of this patient pancreatitis Diagnosis of acute severe pancreatitis mainly depends on persistent organ failure If the patient improves in the next hours and can be extubated, the pancreatitis could be classified as moderate The patient has some predictors of severe acute pancreatitis but right now it cannot be classified as such Atlanta classification defines acute severe pancreatitis as persistent organ failure that lasts more than 48 hours according to Marshall Criteria. Although it’s very likely that this patient behaves as severe, only about 8 hours have elapsed since admission. This question is to illustrate that the diagnosis of acute severe pancreatitis is retrospective and is one of the pitfalls of Atlanta classification. Because of this, patient must be aggressively resuscitated.

After 24 hours in the ICU, the patient is improving. Her current laboratory tests reveal a total bilirubin of 3 mg/dL, conjugated bilirubin of 1.8 mg/dL, unconjugated bilirubin of 1.2 mg/dL, alkaline phosphatase of 200 U/L, AST 150 U/L and ALT 150 U/L. One of the senior attending physicians suggest an early ERCP. What’s the best strategy in the management of this patient biliary tract? Program an early ERCP right now ERCP when patient is discharged from the ICU ERCP if the patient has cholangitis at some point Not perform ERCP under any circumstance Perform a biliary tract percutaneous drainage right now The most powerful indication for an ERCP in the context of acute biliary pancreatitis is the presence of cholangitis. At this point, endoscopic drainage of the biliary tract may be live saving. Also in patients with persistent cholestasis ERCP is an option. Otherwise risk surpasses benefits.

She is discharged to a general ward after 2 days in the ICU. After 2 weeks she appears well but begins with intermittent fever. Laboratory tests show a white blood cell count of 18000/µL. Abdominal CT shows a poorly demarcated area of peripancreatic necrosis near the tail with gas bubbles. Which of the following is the best strategy for the management at this point? Perform a fine needle aspiration biopsy Initiate antibiotics and defer interventional procedures Perform open necrosectomy Perform endoscopic necrosectomy Percutaneous drainage This patient has an acute pancreatic necrosis (not surrounded by a capsule) that has clear imaging signs of infection (gas bubbles) and her clinical state is stable. She can be safely managed with antibiotics. Because the necrosis isn’t

walled off, is highly unlikely to achieve drainage and it’s better to wait until the necrosis become walled-off to intervene. It has been described that early necrosectomy increases mortality and is not an appropriate strategy.

CLINICAL CASE 10 A 70 year old, previously healthy man presents with painless jaundice and itching over the last 2 months. He reports a 10 kg weight loss. Physical examination reveals a thin man. Abdomen is soft but tender to deep palpation in the epigastric area. Laboratory tests report a Total bilirubin of 5 mg/dL, direct bilirubin 4.1 mg/dL, alkaline phosphatase 220 U/L. Abdominal CT shows a 2.8 cm solid mass localized within the pancreatic head with common bile duct dilation. There’s no evidence of vascular invasion. Which of the following is the most appropriate next step in management? Schedule an ERCP Schedule a EUS with FNA Pancreaticoduodenectomy Refer him to medical oncology for chemotherapy Palliative care The standard of care of resectable pancreatic head masses is pancreatoduodenectomy. If the patient doesn’t have cholangitis, it’s preferable not to manipulate the biliary tract before the surgery. Because with CT information the mass is resectable, there’s no need for further local staging tests.

The preliminary report of the surgical pathologist is a malignant lesion but he wants to further analyze the specimen to sign the final diagnosis. Considering epidemiology, which of the following is the most likely diagnoses? Lymphoma Ductal adenocarcinoma Acinar adenocarcinoma IgG4 chronic pancreatitis Retroperitoneal sarcoma The most frequent type of pancreatic cancer is ductal adenocarcinoma.

Which of the following imaging features would not be expected in a patient with your diagnosis? Pancreatic duct dilation Atrophic pancreas Diffuse pancreatic enlargement with sausage appearance Non-enhancing mass Vascular invasion in some patients

Sausage appearance is a typical feature of Ig-G4 chronic pancreatitis.

Which of the following is NOT a typical feature of this disease? It’s one of the most lethal cancers At time of presentation, tumors in the head are smaller than those in the body and tail Only 10-20% are resectable at diagnosis Superior mesenteric artery invasion makes tumor unresectable Common liver duct invasion makes tumor unresectable The resectability of the tumor mainly depends on vascular invasion. In general, is worse when an artery is compromised (it could be celiac axis or superior mesenteric artery). Sometimes there are lesions with portal vein invasion that are amenable to surgery. When there is doubt about the resectability status it’s better to perform an endoscopic ultrasound to clarify this matter.

When this group of patients need medical treatment, which of the following is NOT a usual option? Gemcitabine Oxaliplatin Fluouracil Prednisone Irinotecan Steroids have no role in the management of pancreatic adenocarcinoma.